import work from year 2016-2017
This commit is contained in:
217
3e/Expression_litterale/Periode1/01_entrainement_eval.tex
Normal file
217
3e/Expression_litterale/Periode1/01_entrainement_eval.tex
Normal file
@@ -0,0 +1,217 @@
|
||||
\documentclass[a4paper,10pt]{/media/documents/Cours/Prof/Enseignements/2016-2017/tools/style/classExo}
|
||||
\usepackage{/media/documents/Cours/Prof/Enseignements/2016-2017/theme}
|
||||
%\geometry{left=10mm,right=10mm, top=10mm, bottom=10mm}
|
||||
|
||||
% Title Page
|
||||
\titre{Technique: calcul et évaluation}
|
||||
% \seconde \premiereS \PSTMG \TSTMG
|
||||
\classe{Troisième}
|
||||
\date{Septembre 2016}
|
||||
%\duree{1 heure}
|
||||
\sujet{01}
|
||||
% DS DSCorr DM DMCorr Corr
|
||||
|
||||
%\printanswers
|
||||
\pagestyle{empty}
|
||||
|
||||
\newcommand{\lesexos}{%
|
||||
\begin{Exo}
|
||||
Faire les calculs suivants en détaillant des étapes.
|
||||
\begin{multicols}{3}
|
||||
\begin{enumerate}
|
||||
|
||||
\item $A = -2 - 2 \times ( -3 )$
|
||||
\begin{solution}
|
||||
\begin{eqnarray*}
|
||||
A & = & -2 - 2 \times ( -3 ) \\
|
||||
A & = & -2 + 6 \\
|
||||
A & = & 4
|
||||
\end{eqnarray*}
|
||||
\end{solution}
|
||||
|
||||
\item $B = 1 - ( -4 \times 3 )$
|
||||
\begin{solution}
|
||||
\begin{eqnarray*}
|
||||
A & = & 1 - ( -4 \times 3 ) \\
|
||||
A & = & 1 - ( -12 ) \\
|
||||
A & = & 13
|
||||
\end{eqnarray*}
|
||||
\end{solution}
|
||||
|
||||
\item $C = -10 \times ( -7 ) - 8 \times ( -3 )$
|
||||
\begin{solution}
|
||||
\begin{eqnarray*}
|
||||
A & = & -10 \times ( -7 ) - 8 \times ( -3 ) \\
|
||||
A & = & 70 + 24 \\
|
||||
A & = & 94
|
||||
\end{eqnarray*}
|
||||
\end{solution}
|
||||
\columnbreak
|
||||
|
||||
\item $D = ( 2 - 8 ) \times 7 - 2$
|
||||
\begin{solution}
|
||||
\begin{eqnarray*}
|
||||
A & = & ( 2 - 8 ) \times 7 - 2 \\
|
||||
A & = & -6 \times 7 - 2 \\
|
||||
A & = & -42 - 2 \\
|
||||
A & = & -44
|
||||
\end{eqnarray*}
|
||||
\end{solution}
|
||||
|
||||
\item $E = 8 ( 9 - 10 ) \times ( -2 )$
|
||||
\begin{solution}
|
||||
\begin{eqnarray*}
|
||||
A & = & 8 ( 9 - 10 ) \times ( -2 ) \\
|
||||
A & = & 8 \times ( -1 ) \times ( -2 ) \\
|
||||
A & = & -8 \times ( -2 ) \\
|
||||
A & = & 16
|
||||
\end{eqnarray*}
|
||||
\end{solution}
|
||||
|
||||
\item $F = ( -10 - 2 ) ( 10 - 6 )$
|
||||
\begin{solution}
|
||||
\begin{eqnarray*}
|
||||
F & = & ( -10 - 2 ) ( 10 - 6 ) \\
|
||||
F & = & -12 \times 4 \\
|
||||
F & = & -48
|
||||
\end{eqnarray*}
|
||||
\end{solution}
|
||||
\columnbreak
|
||||
|
||||
\item $G = 5 ( -3 - 8 ) + 3$
|
||||
\begin{solution}
|
||||
\begin{eqnarray*}
|
||||
G & = & 5 ( -3 - 8 ) + 3 \\
|
||||
G & = & 5 \times ( -11 ) + 3 \\
|
||||
G & = & -55 + 3 \\
|
||||
G & = & -52
|
||||
\end{eqnarray*}
|
||||
\end{solution}
|
||||
|
||||
\item $H = 1 - ( -2 - 8 ) \times ( -6 )$
|
||||
\begin{solution}
|
||||
\begin{eqnarray*}
|
||||
H & = & 1 - ( -2 - 8 ) \times ( -6 ) \\
|
||||
H & = & 1 - ( -10 \times ( -6 ) ) \\
|
||||
H & = & 1 - 60 \\
|
||||
H & = & -59
|
||||
\end{eqnarray*}
|
||||
\end{solution}
|
||||
|
||||
\item $I = 5 ( 10 - ( -8 \times 3 ) )$
|
||||
\begin{solution}
|
||||
\begin{eqnarray*}
|
||||
I & = & 5 ( 10 - ( -8 \times 3 ) ) \\
|
||||
I & = & 5 ( 10 - ( -24 ) ) \\
|
||||
I & = & 5 \times 34 \\
|
||||
I & = & 170
|
||||
\end{eqnarray*}
|
||||
\end{solution}
|
||||
\end{enumerate}
|
||||
\end{multicols}
|
||||
\end{Exo}
|
||||
|
||||
\begin{Exo}
|
||||
Évaluer les expressions suivantes
|
||||
|
||||
\begin{multicols}{3}
|
||||
\begin{enumerate}
|
||||
|
||||
|
||||
\item $10 x - 7$ en $x = 3$
|
||||
\begin{solution}
|
||||
\begin{eqnarray*}
|
||||
A & = & 10 \times 3 - 7 \\
|
||||
A & = & 30 - 7 \\
|
||||
A & = & 23
|
||||
\end{eqnarray*}
|
||||
\end{solution}
|
||||
|
||||
|
||||
\item $-3 x - 8$ en $x = -5$
|
||||
\begin{solution}
|
||||
\begin{eqnarray*}
|
||||
A & = & -3 \times ( -5 ) - 8 \\
|
||||
A & = & 15 - 8 \\
|
||||
A & = & 7
|
||||
\end{eqnarray*}
|
||||
\end{solution}
|
||||
|
||||
\item $-3 x - 8$ en $x = 2$
|
||||
\begin{solution}
|
||||
\begin{eqnarray*}
|
||||
A & = & -3 \times 2 - 8 \\
|
||||
A & = & -6 - 8 \\
|
||||
A & = & -14
|
||||
\end{eqnarray*}
|
||||
\end{solution}
|
||||
|
||||
|
||||
|
||||
\item $2 x^{ 2 } + 2 x - 4$ en $x = 4$
|
||||
\begin{solution}
|
||||
\begin{eqnarray*}
|
||||
A & = & 2 \times 4^{ 2 } + 2 \times 4 - 4 \\
|
||||
A & = & 2 \times 16 + 8 - 4 \\
|
||||
A & = & 32 + 8 - 4 \\
|
||||
A & = & 40 - 4 \\
|
||||
A & = & 36
|
||||
\end{eqnarray*}
|
||||
\end{solution}
|
||||
|
||||
\item $2 x^{ 2 } + 2 x - 4$ en $x = 4$
|
||||
\begin{solution}
|
||||
\begin{eqnarray*}
|
||||
A & = & 2 \times 4^{ 2 } + 2 \times 4 - 4 \\
|
||||
A & = & 2 \times 16 + 8 - 4 \\
|
||||
A & = & 32 + 8 - 4 \\
|
||||
A & = & 40 - 4 \\
|
||||
A & = & 36
|
||||
\end{eqnarray*}
|
||||
\end{solution}
|
||||
|
||||
|
||||
\item $-8 x^{ 2 } + 9 x + 8$ en $x = -7$
|
||||
\begin{solution}
|
||||
\begin{eqnarray*}
|
||||
A & = & -8 \times ( -7 )^{ 2 } + 9 \times ( -7 ) + 8 \\
|
||||
A & = & -8 \times 49 - 63 + 8 \\
|
||||
A & = & -392 - 63 + 8 \\
|
||||
A & = & -455 + 8 \\
|
||||
A & = & -447
|
||||
\end{eqnarray*}
|
||||
\end{solution}
|
||||
|
||||
|
||||
\item $-3 x ( x - ( -7 ) )$ en $x = 8$
|
||||
|
||||
\item $-3 x ( x - ( -7 ) )$ en $x = -10$
|
||||
|
||||
|
||||
\item $( -4 x + 5 ) ( -4 - 10 x )$ en $x = 1$
|
||||
\end{enumerate}
|
||||
\end{multicols}
|
||||
\end{Exo}
|
||||
}
|
||||
|
||||
\begin{document}
|
||||
|
||||
\lesexos
|
||||
\setcounter{exo}{0}
|
||||
\vfill
|
||||
|
||||
\lesexos
|
||||
\setcounter{exo}{0}
|
||||
\vfill
|
||||
|
||||
|
||||
\lesexos
|
||||
\setcounter{exo}{0}
|
||||
\vfill
|
||||
|
||||
\end{document}
|
||||
|
||||
%%% Local Variables:
|
||||
%%% mode: latex
|
||||
%%% TeX-master: "master"
|
||||
%%% End:
|
BIN
3e/Expression_litterale/Periode1/QCM.pdf
Normal file
BIN
3e/Expression_litterale/Periode1/QCM.pdf
Normal file
Binary file not shown.
111
3e/Expression_litterale/Periode1/QCM.tex
Normal file
111
3e/Expression_litterale/Periode1/QCM.tex
Normal file
@@ -0,0 +1,111 @@
|
||||
\documentclass[a4paper,10pt,landscape, twocolumn]{/media/documents/Cours/Prof/Enseignements/2016-2017/tools/style/classExo}
|
||||
\usepackage{/media/documents/Cours/Prof/Enseignements/2016-2017/theme}
|
||||
|
||||
% Title Page
|
||||
\titre{Évaluer et égalité - Exercices}
|
||||
% \seconde \premiereS \PSTMG \TSTMG
|
||||
\classe{Troisième}
|
||||
\date{Octobre 2015}
|
||||
|
||||
|
||||
\begin{document}
|
||||
|
||||
\begin{Exo}
|
||||
|
||||
Cet exercice est un questionnaire à choix multiples (QCM). Pour chaque question, une seule des trois réponses proposées est exacte. Sur la copie, indiquer le numéro de la question et recopier, sans justifier, la réponse choisie. Aucun point ne sera enlevé en cas de mauvaise réponse :
|
||||
|
||||
\begin{center}
|
||||
\begin{tabular}{|c|p{5cm}|*{2}{p{2cm}|}p{2.5cm}|}
|
||||
\hline
|
||||
1 & Si $x = 3$ alors $2x(3x + 1)$ est égal à & $9x$ & $3$ & 60\\
|
||||
\hline
|
||||
2 & Si $x = -2$ alors $(6x^2 + 1) - 4$ est égal à & $-2$ & $21$ & $0$\\
|
||||
\hline
|
||||
3 & Si $a = -10$ alors $4a^2 + 9a - 10$ est égal à & $3$ & $300$ & $-10a^2$\\
|
||||
\hline
|
||||
4 & À quelle expression $(x-3)(6x+1)$ est-elle égale? & $3$ & $6x^2 - 3$ & $6x^2-17x-3$ \\
|
||||
\hline
|
||||
5 & À quelle expression $(2x+4)(2x+4)$ est-elle égale? & $(2x+4)^2$ & $2x+4$ & $4x^2 + 16x$ \\
|
||||
\hline
|
||||
6 & $9-49x^2$ est égale à : & $-40x$ & $(3-7x)^2$ & $(3-7x)(3+7x)$\\
|
||||
\hline
|
||||
\end{tabular}
|
||||
\end{center}
|
||||
\end{Exo}
|
||||
|
||||
\begin{Exo}
|
||||
Cet exercice est un QCM (questionnaire à choix multiples).
|
||||
Pour chaque ligne du tableau, une seule affirmation est juste.
|
||||
Sur votre copie, indiquer le numéro de la question et recopier l’affirmation juste.
|
||||
On ne demande pas de justifier.
|
||||
|
||||
\begin{center}
|
||||
\begin{tabular}{|c|p{5cm}|*{3}{p{2cm}|}}
|
||||
\hline
|
||||
& & A & B & C \\
|
||||
\hline
|
||||
1 & $(x-1)^2$ est égale à & $(x-1)(x+1)$ & $x^2 - 2x + 1$ & $x^2 + 2x + 1$ \\
|
||||
\hline
|
||||
2 & $3x^2 + 10x - 5x^2 + 4$ est égale à & $12x$ & $12x^2 + 4$ & $-2x^2 + 10x + 4$ \\
|
||||
\hline
|
||||
3 & $-6x + 3x^2 + 10x - 5x^2 -4x$ est égale à & $-2x^2$ & $-2x$ & $-2$ \\
|
||||
\hline
|
||||
\end{tabular}
|
||||
\end{center}
|
||||
\end{Exo}
|
||||
|
||||
\pagebreak
|
||||
\setcounter{exo}{0}
|
||||
|
||||
|
||||
\begin{Exo}
|
||||
|
||||
Cet exercice est un questionnaire à choix multiples (QCM). Pour chaque question, une seule des trois réponses proposées est exacte. Sur la copie, indiquer le numéro de la question et recopier, sans justifier, la réponse choisie. Aucun point ne sera enlevé en cas de mauvaise réponse :
|
||||
|
||||
\begin{center}
|
||||
\begin{tabular}{|c|p{5cm}|*{2}{p{2cm}|}p{2.5cm}|}
|
||||
\hline
|
||||
1 & Si $x = 3$ alors $2x(3x + 1)$ est égal à & $9x$ & $3$ & 60\\
|
||||
\hline
|
||||
2 & Si $x = -2$ alors $(6x^2 + 1) - 4$ est égal à & $-2$ & $21$ & $0$\\
|
||||
\hline
|
||||
3 & Si $a = -10$ alors $4a^2 + 9a - 10$ est égal à & $3$ & $300$ & $-10a^2$\\
|
||||
\hline
|
||||
4 & À quelle expression $(x-3)(6x+1)$ est-elle égale? & $3$ & $6x^2 - 3$ & $6x^2-17x-3$ \\
|
||||
\hline
|
||||
5 & À quelle expression $(2x+4)(2x+4)$ est-elle égale? & $(2x+4)^2$ & $2x+4$ & $4x^2 + 16x$ \\
|
||||
\hline
|
||||
6 & $9-49x^2$ est égale à : & $-40x$ & $(3-7x)^2$ & $(3-7x)(3+7x)$\\
|
||||
\hline
|
||||
\end{tabular}
|
||||
\end{center}
|
||||
\end{Exo}
|
||||
|
||||
\begin{Exo}
|
||||
Cet exercice est un QCM (questionnaire à choix multiples).
|
||||
Pour chaque ligne du tableau, une seule affirmation est juste.
|
||||
Sur votre copie, indiquer le numéro de la question et recopier l’affirmation juste.
|
||||
On ne demande pas de justifier.
|
||||
|
||||
\begin{center}
|
||||
\begin{tabular}{|c|p{5cm}|*{3}{p{2cm}|}}
|
||||
\hline
|
||||
& & A & B & C \\
|
||||
\hline
|
||||
1 & $(x-1)^2$ est égale à & $(x-1)(x+1)$ & $x^2 - 2x + 1$ & $x^2 + 2x + 1$ \\
|
||||
\hline
|
||||
2 & $3x^2 + 10x - 5x^2 + 4$ est égale à & $12x$ & $12x^2 + 4$ & $-2x^2 + 10x + 4$ \\
|
||||
\hline
|
||||
3 & $-6x + 3x^2 + 10x - 5x^2 -4x$ est égale à & $-2x^2$ & $-2x$ & $-2$ \\
|
||||
\hline
|
||||
\end{tabular}
|
||||
\end{center}
|
||||
\end{Exo}
|
||||
\pagebreak
|
||||
\end{document}
|
||||
|
||||
%%% Local Variables:
|
||||
%%% mode: latex
|
||||
%%% TeX-master: "master"
|
||||
%%% End:
|
||||
|
BIN
3e/Expression_litterale/Periode1/all_entrainement_eval.pdf
Normal file
BIN
3e/Expression_litterale/Periode1/all_entrainement_eval.pdf
Normal file
Binary file not shown.
BIN
3e/Expression_litterale/Periode1/carre_pierres.pdf
Normal file
BIN
3e/Expression_litterale/Periode1/carre_pierres.pdf
Normal file
Binary file not shown.
626
3e/Expression_litterale/Periode1/carre_pierres.svg
Normal file
626
3e/Expression_litterale/Periode1/carre_pierres.svg
Normal file
@@ -0,0 +1,626 @@
|
||||
<?xml version="1.0" encoding="UTF-8" standalone="no"?>
|
||||
<!-- Created with Inkscape (http://www.inkscape.org/) -->
|
||||
|
||||
<svg
|
||||
xmlns:dc="http://purl.org/dc/elements/1.1/"
|
||||
xmlns:cc="http://creativecommons.org/ns#"
|
||||
xmlns:rdf="http://www.w3.org/1999/02/22-rdf-syntax-ns#"
|
||||
xmlns:svg="http://www.w3.org/2000/svg"
|
||||
xmlns="http://www.w3.org/2000/svg"
|
||||
xmlns:xlink="http://www.w3.org/1999/xlink"
|
||||
xmlns:sodipodi="http://sodipodi.sourceforge.net/DTD/sodipodi-0.dtd"
|
||||
xmlns:inkscape="http://www.inkscape.org/namespaces/inkscape"
|
||||
id="svg2"
|
||||
version="1.1"
|
||||
inkscape:version="0.91 r13725"
|
||||
xml:space="preserve"
|
||||
width="743.75"
|
||||
height="1052.5"
|
||||
viewBox="0 0 743.75 1052.5"
|
||||
sodipodi:docname="carre_pierres.svg"><metadata
|
||||
id="metadata8"><rdf:RDF><cc:Work
|
||||
rdf:about=""><dc:format>image/svg+xml</dc:format><dc:type
|
||||
rdf:resource="http://purl.org/dc/dcmitype/StillImage" /><dc:title></dc:title></cc:Work></rdf:RDF></metadata><defs
|
||||
id="defs6"><clipPath
|
||||
clipPathUnits="userSpaceOnUse"
|
||||
id="clipPath52"><path
|
||||
d="m 4737,389.668 221,0 0,-49 -221,0 0,49 z"
|
||||
id="path54"
|
||||
inkscape:connector-curvature="0" /></clipPath><clipPath
|
||||
clipPathUnits="userSpaceOnUse"
|
||||
id="clipPath376"><path
|
||||
d="m 598,4093.67 0,-103 169,0 0,103 -10,0 0,-96 -149,0 0,96 -10,0 z"
|
||||
id="path378"
|
||||
inkscape:connector-curvature="0"
|
||||
style="clip-rule:evenodd" /></clipPath><clipPath
|
||||
clipPathUnits="userSpaceOnUse"
|
||||
id="clipPath394"><path
|
||||
d="m 771,4093.67 0,-103 469,0 0,103 -10,0 0,-96 -448,0 0,96 -11,0 z"
|
||||
id="path396"
|
||||
inkscape:connector-curvature="0"
|
||||
style="clip-rule:evenodd" /></clipPath><clipPath
|
||||
clipPathUnits="userSpaceOnUse"
|
||||
id="clipPath412"><path
|
||||
d="m 1244,4093.67 0,-103 468,0 0,103 -10,0 0,-96 -448,0 0,96 -10,0 z"
|
||||
id="path414"
|
||||
inkscape:connector-curvature="0"
|
||||
style="clip-rule:evenodd" /></clipPath><clipPath
|
||||
clipPathUnits="userSpaceOnUse"
|
||||
id="clipPath430"><path
|
||||
d="m 1716,4093.67 0,-103 477,0 0,103 -10,0 0,-96 -456,0 0,96 -11,0 z"
|
||||
id="path432"
|
||||
inkscape:connector-curvature="0"
|
||||
style="clip-rule:evenodd" /></clipPath><clipPath
|
||||
clipPathUnits="userSpaceOnUse"
|
||||
id="clipPath446"><path
|
||||
d="m 2678,4093.67 0,-103 171,0 0,103 -10,0 0,-96 -150,0 0,96 -11,0 z"
|
||||
id="path448"
|
||||
inkscape:connector-curvature="0"
|
||||
style="clip-rule:evenodd" /></clipPath><clipPath
|
||||
clipPathUnits="userSpaceOnUse"
|
||||
id="clipPath464"><path
|
||||
d="m 2853,4093.67 0,-103 477,0 0,103 -10,0 0,-96 -456,0 0,96 -11,0 z"
|
||||
id="path466"
|
||||
inkscape:connector-curvature="0"
|
||||
style="clip-rule:evenodd" /></clipPath><clipPath
|
||||
clipPathUnits="userSpaceOnUse"
|
||||
id="clipPath482"><path
|
||||
d="m 3334,4093.67 0,-103 477,0 0,103 -11,0 0,-96 -456,0 0,96 -10,0 z"
|
||||
id="path484"
|
||||
inkscape:connector-curvature="0"
|
||||
style="clip-rule:evenodd" /></clipPath><clipPath
|
||||
clipPathUnits="userSpaceOnUse"
|
||||
id="clipPath500"><path
|
||||
d="m 3815,4093.67 0,-103 547,0 0,103 -10,0 0,-96 -527,0 0,96 -10,0 z"
|
||||
id="path502"
|
||||
inkscape:connector-curvature="0"
|
||||
style="clip-rule:evenodd" /></clipPath><clipPath
|
||||
clipPathUnits="userSpaceOnUse"
|
||||
id="clipPath582"><path
|
||||
d="m 598,3986.67 0,-103 169,0 0,103 -10,0 0,-96 -149,0 0,96 -10,0 z"
|
||||
id="path584"
|
||||
inkscape:connector-curvature="0"
|
||||
style="clip-rule:evenodd" /></clipPath><clipPath
|
||||
clipPathUnits="userSpaceOnUse"
|
||||
id="clipPath606"><path
|
||||
d="m 2678,3986.67 0,-103 171,0 0,103 -10,0 0,-96 -150,0 0,96 -11,0 z"
|
||||
id="path608"
|
||||
inkscape:connector-curvature="0"
|
||||
style="clip-rule:evenodd" /></clipPath><clipPath
|
||||
clipPathUnits="userSpaceOnUse"
|
||||
id="clipPath686"><path
|
||||
d="m 598,3879.67 0,-103 169,0 0,103 -10,0 0,-96 -149,0 0,96 -10,0 z"
|
||||
id="path688"
|
||||
inkscape:connector-curvature="0"
|
||||
style="clip-rule:evenodd" /></clipPath><clipPath
|
||||
clipPathUnits="userSpaceOnUse"
|
||||
id="clipPath710"><path
|
||||
d="m 2678,3879.67 0,-103 171,0 0,103 -10,0 0,-96 -150,0 0,96 -11,0 z"
|
||||
id="path712"
|
||||
inkscape:connector-curvature="0"
|
||||
style="clip-rule:evenodd" /></clipPath><clipPath
|
||||
clipPathUnits="userSpaceOnUse"
|
||||
id="clipPath790"><path
|
||||
d="m 598,3772.67 0,-103 169,0 0,103 -10,0 0,-96 -149,0 0,96 -10,0 z"
|
||||
id="path792"
|
||||
inkscape:connector-curvature="0"
|
||||
style="clip-rule:evenodd" /></clipPath><clipPath
|
||||
clipPathUnits="userSpaceOnUse"
|
||||
id="clipPath814"><path
|
||||
d="m 2678,3772.67 0,-103 171,0 0,103 -10,0 0,-96 -150,0 0,96 -11,0 z"
|
||||
id="path816"
|
||||
inkscape:connector-curvature="0"
|
||||
style="clip-rule:evenodd" /></clipPath><clipPath
|
||||
clipPathUnits="userSpaceOnUse"
|
||||
id="clipPath894"><path
|
||||
d="m 598,3665.67 0,-103 169,0 0,103 -10,0 0,-96 -149,0 0,96 -10,0 z"
|
||||
id="path896"
|
||||
inkscape:connector-curvature="0"
|
||||
style="clip-rule:evenodd" /></clipPath><clipPath
|
||||
clipPathUnits="userSpaceOnUse"
|
||||
id="clipPath918"><path
|
||||
d="m 2678,3665.67 0,-103 171,0 0,103 -10,0 0,-96 -150,0 0,96 -11,0 z"
|
||||
id="path920"
|
||||
inkscape:connector-curvature="0"
|
||||
style="clip-rule:evenodd" /></clipPath><clipPath
|
||||
clipPathUnits="userSpaceOnUse"
|
||||
id="clipPath998"><path
|
||||
d="m 598,3558.67 0,-104 169,0 0,104 -10,0 0,-96 -149,0 0,96 -10,0 z"
|
||||
id="path1000"
|
||||
inkscape:connector-curvature="0"
|
||||
style="clip-rule:evenodd" /></clipPath><clipPath
|
||||
clipPathUnits="userSpaceOnUse"
|
||||
id="clipPath1022"><path
|
||||
d="m 2678,3558.67 0,-104 171,0 0,104 -10,0 0,-96 -150,0 0,96 -11,0 z"
|
||||
id="path1024"
|
||||
inkscape:connector-curvature="0"
|
||||
style="clip-rule:evenodd" /></clipPath><clipPath
|
||||
clipPathUnits="userSpaceOnUse"
|
||||
id="clipPath1102"><path
|
||||
d="m 598,3450.67 0,-103 169,0 0,103 -10,0 0,-95 -149,0 0,95 -10,0 z"
|
||||
id="path1104"
|
||||
inkscape:connector-curvature="0"
|
||||
style="clip-rule:evenodd" /></clipPath><clipPath
|
||||
clipPathUnits="userSpaceOnUse"
|
||||
id="clipPath1126"><path
|
||||
d="m 2678,3450.67 0,-103 171,0 0,103 -10,0 0,-95 -150,0 0,95 -11,0 z"
|
||||
id="path1128"
|
||||
inkscape:connector-curvature="0"
|
||||
style="clip-rule:evenodd" /></clipPath><clipPath
|
||||
clipPathUnits="userSpaceOnUse"
|
||||
id="clipPath1206"><path
|
||||
d="m 598,3343.67 0,-103 169,0 0,103 -10,0 0,-95 -149,0 0,95 -10,0 z"
|
||||
id="path1208"
|
||||
inkscape:connector-curvature="0"
|
||||
style="clip-rule:evenodd" /></clipPath><clipPath
|
||||
clipPathUnits="userSpaceOnUse"
|
||||
id="clipPath1230"><path
|
||||
d="m 2678,3343.67 0,-103 171,0 0,103 -10,0 0,-95 -150,0 0,95 -11,0 z"
|
||||
id="path1232"
|
||||
inkscape:connector-curvature="0"
|
||||
style="clip-rule:evenodd" /></clipPath><clipPath
|
||||
clipPathUnits="userSpaceOnUse"
|
||||
id="clipPath1310"><path
|
||||
d="m 598,3236.67 0,-103 169,0 0,103 -10,0 0,-96 -149,0 0,96 -10,0 z"
|
||||
id="path1312"
|
||||
inkscape:connector-curvature="0"
|
||||
style="clip-rule:evenodd" /></clipPath><clipPath
|
||||
clipPathUnits="userSpaceOnUse"
|
||||
id="clipPath1330"><path
|
||||
d="m 2678,3236.67 0,-103 171,0 0,103 -10,0 0,-96 -150,0 0,96 -11,0 z"
|
||||
id="path1332"
|
||||
inkscape:connector-curvature="0"
|
||||
style="clip-rule:evenodd" /></clipPath><clipPath
|
||||
clipPathUnits="userSpaceOnUse"
|
||||
id="clipPath1410"><path
|
||||
d="m 598,3129.67 0,-103 169,0 0,103 -10,0 0,-96 -149,0 0,96 -10,0 z"
|
||||
id="path1412"
|
||||
inkscape:connector-curvature="0"
|
||||
style="clip-rule:evenodd" /></clipPath><clipPath
|
||||
clipPathUnits="userSpaceOnUse"
|
||||
id="clipPath1430"><path
|
||||
d="m 2678,3129.67 0,-103 171,0 0,103 -10,0 0,-96 -150,0 0,96 -11,0 z"
|
||||
id="path1432"
|
||||
inkscape:connector-curvature="0"
|
||||
style="clip-rule:evenodd" /></clipPath><clipPath
|
||||
clipPathUnits="userSpaceOnUse"
|
||||
id="clipPath1510"><path
|
||||
d="m 728,3023.67 39,0 0,-96 -39,0 0,96 z"
|
||||
id="path1512"
|
||||
inkscape:connector-curvature="0" /></clipPath><clipPath
|
||||
clipPathUnits="userSpaceOnUse"
|
||||
id="clipPath1520"><path
|
||||
d="m 598,3022.67 0,-103 169,0 0,103 -10,0 0,-96 -149,0 0,96 -10,0 z"
|
||||
id="path1522"
|
||||
inkscape:connector-curvature="0"
|
||||
style="clip-rule:evenodd" /></clipPath><clipPath
|
||||
clipPathUnits="userSpaceOnUse"
|
||||
id="clipPath1540"><path
|
||||
d="m 2810,3023.67 39,0 0,-96 -39,0 0,96 z"
|
||||
id="path1542"
|
||||
inkscape:connector-curvature="0" /></clipPath><clipPath
|
||||
clipPathUnits="userSpaceOnUse"
|
||||
id="clipPath1550"><path
|
||||
d="m 2678,3022.67 0,-103 171,0 0,103 -10,0 0,-96 -150,0 0,96 -11,0 z"
|
||||
id="path1552"
|
||||
inkscape:connector-curvature="0"
|
||||
style="clip-rule:evenodd" /></clipPath></defs><sodipodi:namedview
|
||||
pagecolor="#ffffff"
|
||||
bordercolor="#666666"
|
||||
borderopacity="1"
|
||||
objecttolerance="10"
|
||||
gridtolerance="10"
|
||||
guidetolerance="10"
|
||||
inkscape:pageopacity="0"
|
||||
inkscape:pageshadow="2"
|
||||
inkscape:window-width="1366"
|
||||
inkscape:window-height="740"
|
||||
id="namedview4"
|
||||
showgrid="false"
|
||||
inkscape:zoom="0.63421264"
|
||||
inkscape:cx="199.9524"
|
||||
inkscape:cy="547.38179"
|
||||
inkscape:window-x="0"
|
||||
inkscape:window-y="0"
|
||||
inkscape:window-maximized="1"
|
||||
inkscape:current-layer="g10" /><g
|
||||
id="g10"
|
||||
inkscape:groupmode="layer"
|
||||
inkscape:label="exercices12"
|
||||
transform="matrix(1.25,0,0,-1.25,0,1052.5)"><g
|
||||
id="g48"
|
||||
transform="scale(0.12,0.12)"><g
|
||||
clip-path="url(#clipPath52)"
|
||||
id="g50"><g
|
||||
transform="scale(8.33333,8.33333)"
|
||||
id="g56" /></g></g><g
|
||||
id="g366" /><g
|
||||
id="g370" /><g
|
||||
id="g436" /><g
|
||||
id="g440" /><g
|
||||
id="g1316" /><g
|
||||
id="g1416" /><g
|
||||
id="g1506"
|
||||
transform="scale(0.12,0.12)"><g
|
||||
clip-path="url(#clipPath1510)"
|
||||
id="g1508"><g
|
||||
transform="scale(8.33333,8.33333)"
|
||||
id="g1514" /></g></g><g
|
||||
id="g1526" /><g
|
||||
id="g1536"
|
||||
transform="scale(0.12,0.12)"><g
|
||||
clip-path="url(#clipPath1540)"
|
||||
id="g1538"><g
|
||||
transform="scale(8.33333,8.33333)"
|
||||
id="g1544" /></g></g><flowRoot
|
||||
xml:space="preserve"
|
||||
id="flowRoot6902"
|
||||
style="fill:black;stroke:none;stroke-opacity:1;stroke-width:1px;stroke-linejoin:miter;stroke-linecap:butt;fill-opacity:1;font-family:sans-serif;font-style:normal;font-weight:normal;font-size:15px;line-height:125%;letter-spacing:0px;word-spacing:0px"><flowRegion
|
||||
id="flowRegion6904"><rect
|
||||
id="rect6906"
|
||||
width="327.79132"
|
||||
height="276.50424"
|
||||
x="43.482521"
|
||||
y="42.367584" /></flowRegion><flowPara
|
||||
id="flowPara6908"></flowPara></flowRoot><g
|
||||
id="g7710"
|
||||
transform="translate(4.459824,-421.51008)"><g
|
||||
transform="translate(282.14346,83.916164)"
|
||||
id="g6820"><path
|
||||
inkscape:connector-curvature="0"
|
||||
id="path1682"
|
||||
style="fill:#000000;fill-opacity:1;fill-rule:nonzero;stroke:none"
|
||||
d="m 113.36046,746.35388 0,0.24 -63.24,0 0,-0.6 63.24,0 0,0.36" /><path
|
||||
inkscape:connector-curvature="0"
|
||||
id="path1684"
|
||||
style="fill:#000000;fill-opacity:1;fill-rule:nonzero;stroke:none"
|
||||
d="m 113.36046,682.99388 0.36,0 0,63.36 -0.72,0 0,-63.36 0.36,0" /><path
|
||||
inkscape:connector-curvature="0"
|
||||
id="path1686"
|
||||
style="fill:#000000;fill-opacity:1;fill-rule:nonzero;stroke:none"
|
||||
d="m 50.12046,682.99388 0,-0.24 63.24,0 0,0.6 -63.24,0 0,-0.36" /><path
|
||||
inkscape:connector-curvature="0"
|
||||
id="path1688"
|
||||
style="fill:#000000;fill-opacity:1;fill-rule:nonzero;stroke:none"
|
||||
d="m 50.12046,746.35388 -0.36,0 0,-63.36 0.72,0 0,63.36 -0.36,0" /><path
|
||||
inkscape:connector-curvature="0"
|
||||
id="path1690"
|
||||
style="fill:#000000;fill-opacity:1;fill-rule:nonzero;stroke:none"
|
||||
d="m 113.36046,733.63388 0,0.36 -63.24,0 0,-0.6 63.24,0 0,0.24" /><path
|
||||
inkscape:connector-curvature="0"
|
||||
id="path1692"
|
||||
style="fill:#000000;fill-opacity:1;fill-rule:nonzero;stroke:none"
|
||||
d="m 100.76046,682.99388 0.24,0 0,63.36 -0.6,0 0,-63.36 0.36,0" /><path
|
||||
inkscape:connector-curvature="0"
|
||||
id="path1694"
|
||||
style="fill:#000000;fill-opacity:1;fill-rule:nonzero;stroke:none"
|
||||
d="m 113.36046,695.71388 0,0.36 -63.24,0 0,-0.72 63.24,0 0,0.36" /><path
|
||||
inkscape:connector-curvature="0"
|
||||
id="path1696"
|
||||
style="fill:#000000;fill-opacity:1;fill-rule:nonzero;stroke:none"
|
||||
d="m 62.72046,682.99388 0.36,0 0,63.36 -0.6,0 0,-63.36 0.24,0" /><path
|
||||
inkscape:connector-curvature="0"
|
||||
id="path1698"
|
||||
style="fill:#000000;fill-opacity:1;fill-rule:nonzero;stroke:none"
|
||||
d="m 75.44046,733.63388 0.24,0 0,12.72 -0.6,0 0,-12.72 0.36,0" /><path
|
||||
inkscape:connector-curvature="0"
|
||||
id="path1700"
|
||||
style="fill:#000000;fill-opacity:1;fill-rule:nonzero;stroke:none"
|
||||
d="m 88.04046,733.63388 0.24,0 0,12.72 -0.48,0 0,-12.72 0.24,0" /><path
|
||||
inkscape:connector-curvature="0"
|
||||
id="path1702"
|
||||
style="fill:#000000;fill-opacity:1;fill-rule:nonzero;stroke:none"
|
||||
d="m 113.36046,721.03388 0,0.36 -12.6,0 0,-0.72 12.6,0 0,0.36" /><path
|
||||
inkscape:connector-curvature="0"
|
||||
id="path1704"
|
||||
style="fill:#000000;fill-opacity:1;fill-rule:nonzero;stroke:none"
|
||||
d="m 113.36046,708.31388 0,0.36 -12.6,0 0,-0.6 12.6,0 0,0.24" /><path
|
||||
inkscape:connector-curvature="0"
|
||||
id="path1706"
|
||||
style="fill:#000000;fill-opacity:1;fill-rule:nonzero;stroke:none"
|
||||
d="m 88.04046,682.99388 0.24,0 0,12.72 -0.48,0 0,-12.72 0.24,0" /><path
|
||||
inkscape:connector-curvature="0"
|
||||
id="path1708"
|
||||
style="fill:#000000;fill-opacity:1;fill-rule:nonzero;stroke:none"
|
||||
d="m 75.44046,682.99388 0.24,0 0,12.72 -0.6,0 0,-12.72 0.36,0" /><path
|
||||
inkscape:connector-curvature="0"
|
||||
id="path1710"
|
||||
style="fill:#000000;fill-opacity:1;fill-rule:nonzero;stroke:none"
|
||||
d="m 50.12046,708.31388 0,-0.24 12.6,0 0,0.6 -12.6,0 0,-0.36" /><path
|
||||
inkscape:connector-curvature="0"
|
||||
id="path1712"
|
||||
style="fill:#000000;fill-opacity:1;fill-rule:nonzero;stroke:none"
|
||||
d="m 50.12046,721.03388 0,-0.36 12.6,0 0,0.72 -12.6,0 0,-0.36" /></g><g
|
||||
transform="translate(285.71126,41.841439)"
|
||||
id="g6838"><path
|
||||
inkscape:connector-curvature="0"
|
||||
id="path1714"
|
||||
style="fill:#000000;fill-opacity:1;fill-rule:nonzero;stroke:none"
|
||||
d="m 151.28046,759.07388 0,-0.36 126.36,0 0,0.6 -126.36,0 0,-0.24" /><path
|
||||
inkscape:connector-curvature="0"
|
||||
id="path1716"
|
||||
style="fill:#000000;fill-opacity:1;fill-rule:nonzero;stroke:none"
|
||||
d="m 151.28046,632.35388 0.24,0 0,126.72 -0.6,0 0,-126.72 0.36,0" /><path
|
||||
inkscape:connector-curvature="0"
|
||||
id="path1718"
|
||||
style="fill:#000000;fill-opacity:1;fill-rule:nonzero;stroke:none"
|
||||
d="m 277.64046,632.35388 0,0.24 -126.36,0 0,-0.6 126.36,0 0,0.36" /><path
|
||||
inkscape:connector-curvature="0"
|
||||
id="path1720"
|
||||
style="fill:#000000;fill-opacity:1;fill-rule:nonzero;stroke:none"
|
||||
d="m 277.64046,759.07388 -0.24,0 0,-126.72 0.6,0 0,126.72 -0.36,0" /><path
|
||||
inkscape:connector-curvature="0"
|
||||
id="path1722"
|
||||
style="fill:#000000;fill-opacity:1;fill-rule:nonzero;stroke:none"
|
||||
d="m 277.64046,746.35388 0,0.24 -126.36,0 0,-0.6 126.36,0 0,0.36" /><path
|
||||
inkscape:connector-curvature="0"
|
||||
id="path1724"
|
||||
style="fill:#000000;fill-opacity:1;fill-rule:nonzero;stroke:none"
|
||||
d="m 265.04046,632.35388 0.36,0 0,126.72 -0.72,0 0,-126.72 0.36,0" /><path
|
||||
inkscape:connector-curvature="0"
|
||||
id="path1726"
|
||||
style="fill:#000000;fill-opacity:1;fill-rule:nonzero;stroke:none"
|
||||
d="m 277.64046,644.95388 0,0.36 -126.36,0 0,-0.72 126.36,0 0,0.36" /><path
|
||||
inkscape:connector-curvature="0"
|
||||
id="path1728"
|
||||
style="fill:#000000;fill-opacity:1;fill-rule:nonzero;stroke:none"
|
||||
d="m 164.00046,632.35388 0.24,0 0,126.72 -0.6,0 0,-126.72 0.36,0" /><path
|
||||
inkscape:connector-curvature="0"
|
||||
id="path1730"
|
||||
style="fill:#000000;fill-opacity:1;fill-rule:nonzero;stroke:none"
|
||||
d="m 164.00046,733.63388 0,0.36 -12.72,0 0,-0.6 12.72,0 0,0.24" /><path
|
||||
inkscape:connector-curvature="0"
|
||||
id="path1732"
|
||||
style="fill:#000000;fill-opacity:1;fill-rule:nonzero;stroke:none"
|
||||
d="m 164.00046,721.03388 0,0.36 -12.72,0 0,-0.72 12.72,0 0,0.36" /><path
|
||||
inkscape:connector-curvature="0"
|
||||
id="path1734"
|
||||
style="fill:#000000;fill-opacity:1;fill-rule:nonzero;stroke:none"
|
||||
d="m 164.00046,708.31388 0,0.36 -12.72,0 0,-0.6 12.72,0 0,0.24" /><path
|
||||
inkscape:connector-curvature="0"
|
||||
id="path1736"
|
||||
style="fill:#000000;fill-opacity:1;fill-rule:nonzero;stroke:none"
|
||||
d="m 164.00046,695.71388 0,0.36 -12.72,0 0,-0.72 12.72,0 0,0.36" /><path
|
||||
inkscape:connector-curvature="0"
|
||||
id="path1738"
|
||||
style="fill:#000000;fill-opacity:1;fill-rule:nonzero;stroke:none"
|
||||
d="m 164.00046,682.99388 0,0.36 -12.72,0 0,-0.6 12.72,0 0,0.24" /><path
|
||||
inkscape:connector-curvature="0"
|
||||
id="path1740"
|
||||
style="fill:#000000;fill-opacity:1;fill-rule:nonzero;stroke:none"
|
||||
d="m 164.00046,670.27388 0,0.36 -12.72,0 0,-0.72 12.72,0 0,0.36" /><path
|
||||
inkscape:connector-curvature="0"
|
||||
id="path1742"
|
||||
style="fill:#000000;fill-opacity:1;fill-rule:nonzero;stroke:none"
|
||||
d="m 164.00046,657.67388 0,0.24 -12.72,0 0,-0.6 12.72,0 0,0.36" /><path
|
||||
inkscape:connector-curvature="0"
|
||||
id="path1744"
|
||||
style="fill:#000000;fill-opacity:1;fill-rule:nonzero;stroke:none"
|
||||
d="m 176.60046,746.35388 0.36,0 0,12.72 -0.72,0 0,-12.72 0.36,0" /><path
|
||||
inkscape:connector-curvature="0"
|
||||
id="path1746"
|
||||
style="fill:#000000;fill-opacity:1;fill-rule:nonzero;stroke:none"
|
||||
d="m 189.20046,746.35388 0.36,0 0,12.72 -0.72,0 0,-12.72 0.36,0" /><path
|
||||
inkscape:connector-curvature="0"
|
||||
id="path1748"
|
||||
style="fill:#000000;fill-opacity:1;fill-rule:nonzero;stroke:none"
|
||||
d="m 201.80046,746.35388 0.36,0 0,12.72 -0.6,0 0,-12.72 0.24,0" /><path
|
||||
inkscape:connector-curvature="0"
|
||||
id="path1750"
|
||||
style="fill:#000000;fill-opacity:1;fill-rule:nonzero;stroke:none"
|
||||
d="m 214.40046,746.35388 0.36,0 0,12.72 -0.6,0 0,-12.72 0.24,0" /><path
|
||||
inkscape:connector-curvature="0"
|
||||
id="path1752"
|
||||
style="fill:#000000;fill-opacity:1;fill-rule:nonzero;stroke:none"
|
||||
d="m 227.24046,746.35388 0.12,0 0,12.72 -0.48,0 0,-12.72 0.36,0" /><path
|
||||
inkscape:connector-curvature="0"
|
||||
id="path1754"
|
||||
style="fill:#000000;fill-opacity:1;fill-rule:nonzero;stroke:none"
|
||||
d="m 239.84046,746.35388 0.24,0 0,12.72 -0.6,0 0,-12.72 0.36,0" /><path
|
||||
inkscape:connector-curvature="0"
|
||||
id="path1756"
|
||||
style="fill:#000000;fill-opacity:1;fill-rule:nonzero;stroke:none"
|
||||
d="m 252.44046,746.35388 0.36,0 0,12.72 -0.72,0 0,-12.72 0.36,0" /><path
|
||||
inkscape:connector-curvature="0"
|
||||
id="path1758"
|
||||
style="fill:#000000;fill-opacity:1;fill-rule:nonzero;stroke:none"
|
||||
d="m 277.64046,733.63388 0,0.36 -12.6,0 0,-0.6 12.6,0 0,0.24" /><path
|
||||
inkscape:connector-curvature="0"
|
||||
id="path1760"
|
||||
style="fill:#000000;fill-opacity:1;fill-rule:nonzero;stroke:none"
|
||||
d="m 277.64046,721.03388 0,0.36 -12.6,0 0,-0.72 12.6,0 0,0.36" /><path
|
||||
inkscape:connector-curvature="0"
|
||||
id="path1762"
|
||||
style="fill:#000000;fill-opacity:1;fill-rule:nonzero;stroke:none"
|
||||
d="m 277.64046,708.31388 0,0.36 -12.6,0 0,-0.6 12.6,0 0,0.24" /><path
|
||||
inkscape:connector-curvature="0"
|
||||
id="path1764"
|
||||
style="fill:#000000;fill-opacity:1;fill-rule:nonzero;stroke:none"
|
||||
d="m 277.64046,695.71388 0,0.36 -12.6,0 0,-0.72 12.6,0 0,0.36" /><path
|
||||
inkscape:connector-curvature="0"
|
||||
id="path1766"
|
||||
style="fill:#000000;fill-opacity:1;fill-rule:nonzero;stroke:none"
|
||||
d="m 277.64046,682.99388 0,0.36 -12.6,0 0,-0.6 12.6,0 0,0.24" /><path
|
||||
inkscape:connector-curvature="0"
|
||||
id="path1768"
|
||||
style="fill:#000000;fill-opacity:1;fill-rule:nonzero;stroke:none"
|
||||
d="m 277.64046,670.27388 0,0.36 -12.6,0 0,-0.72 12.6,0 0,0.36" /><path
|
||||
inkscape:connector-curvature="0"
|
||||
id="path1770"
|
||||
style="fill:#000000;fill-opacity:1;fill-rule:nonzero;stroke:none"
|
||||
d="m 277.64046,657.67388 0,0.24 -12.6,0 0,-0.6 12.6,0 0,0.36" /><path
|
||||
inkscape:connector-curvature="0"
|
||||
id="path1772"
|
||||
style="fill:#000000;fill-opacity:1;fill-rule:nonzero;stroke:none"
|
||||
d="m 252.44046,632.35388 0.36,0 0,12.6 -0.72,0 0,-12.6 0.36,0" /><path
|
||||
inkscape:connector-curvature="0"
|
||||
id="path1774"
|
||||
style="fill:#000000;fill-opacity:1;fill-rule:nonzero;stroke:none"
|
||||
d="m 239.84046,632.35388 0.24,0 0,12.6 -0.6,0 0,-12.6 0.36,0" /><path
|
||||
inkscape:connector-curvature="0"
|
||||
id="path1776"
|
||||
style="fill:#000000;fill-opacity:1;fill-rule:nonzero;stroke:none"
|
||||
d="m 227.24046,632.35388 0.12,0 0,12.6 -0.48,0 0,-12.6 0.36,0" /><path
|
||||
inkscape:connector-curvature="0"
|
||||
id="path1778"
|
||||
style="fill:#000000;fill-opacity:1;fill-rule:nonzero;stroke:none"
|
||||
d="m 214.40046,632.35388 0.36,0 0,12.6 -0.6,0 0,-12.6 0.24,0" /><path
|
||||
inkscape:connector-curvature="0"
|
||||
id="path1780"
|
||||
style="fill:#000000;fill-opacity:1;fill-rule:nonzero;stroke:none"
|
||||
d="m 201.80046,632.35388 0.36,0 0,12.6 -0.6,0 0,-12.6 0.24,0" /><path
|
||||
inkscape:connector-curvature="0"
|
||||
id="path1782"
|
||||
style="fill:#000000;fill-opacity:1;fill-rule:nonzero;stroke:none"
|
||||
d="m 189.20046,632.35388 0.36,0 0,12.6 -0.72,0 0,-12.6 0.36,0" /><path
|
||||
inkscape:connector-curvature="0"
|
||||
id="path1784"
|
||||
style="fill:#000000;fill-opacity:1;fill-rule:nonzero;stroke:none"
|
||||
d="m 176.60046,632.35388 0.36,0 0,12.6 -0.72,0 0,-12.6 0.36,0" /></g><g
|
||||
transform="translate(268.76422,85.82676)"
|
||||
id="g6876"><path
|
||||
inkscape:connector-curvature="0"
|
||||
id="path1786"
|
||||
style="fill:#000000;fill-opacity:1;fill-rule:nonzero;stroke:none"
|
||||
d="m 138.56046,657.67388 0,0.24 -88.44,0 0,-0.6 88.44,0 0,0.36" /><path
|
||||
inkscape:connector-curvature="0"
|
||||
id="path1788"
|
||||
style="fill:#000000;fill-opacity:1;fill-rule:nonzero;stroke:none"
|
||||
d="m 138.56046,568.87364 0.36,0 0,88.80024 -0.6,0 0,-88.80024 0.24,0" /><path
|
||||
inkscape:connector-curvature="0"
|
||||
id="path1790"
|
||||
style="fill:#000000;fill-opacity:1;fill-rule:nonzero;stroke:none"
|
||||
d="m 50.12046,568.87364 0,-0.24 88.44,0 0,0.6 -88.44,0 0,-0.36" /><path
|
||||
inkscape:connector-curvature="0"
|
||||
id="path1792"
|
||||
style="fill:#000000;fill-opacity:1;fill-rule:nonzero;stroke:none"
|
||||
d="m 50.12046,657.67388 -0.36,0 0,-88.80024 0.72,0 0,88.80024 -0.36,0" /><path
|
||||
inkscape:connector-curvature="0"
|
||||
id="path1794"
|
||||
style="fill:#000000;fill-opacity:1;fill-rule:nonzero;stroke:none"
|
||||
d="m 138.56046,644.95388 0,0.36 -88.44,0 0,-0.72 88.44,0 0,0.36" /><path
|
||||
inkscape:connector-curvature="0"
|
||||
id="path1796"
|
||||
style="fill:#000000;fill-opacity:1;fill-rule:nonzero;stroke:none"
|
||||
d="m 125.96046,568.87364 0.36,0 0,88.80024 -0.6,0 0,-88.80024 0.24,0" /><path
|
||||
inkscape:connector-curvature="0"
|
||||
id="path1798"
|
||||
style="fill:#000000;fill-opacity:1;fill-rule:nonzero;stroke:none"
|
||||
d="m 50.12046,581.59364 0,-0.36 88.44,0 0,0.72 -88.44,0 0,-0.36" /><path
|
||||
inkscape:connector-curvature="0"
|
||||
id="path1800"
|
||||
style="fill:#000000;fill-opacity:1;fill-rule:nonzero;stroke:none"
|
||||
d="m 62.72046,568.87364 0.36,0 0,88.80024 -0.6,0 0,-88.80024 0.24,0" /><path
|
||||
inkscape:connector-curvature="0"
|
||||
id="path1802"
|
||||
style="fill:#000000;fill-opacity:1;fill-rule:nonzero;stroke:none"
|
||||
d="m 75.44046,644.95388 0.24,0 0,12.72 -0.6,0 0,-12.72 0.36,0" /><path
|
||||
inkscape:connector-curvature="0"
|
||||
id="path1804"
|
||||
style="fill:#000000;fill-opacity:1;fill-rule:nonzero;stroke:none"
|
||||
d="m 88.04046,644.95388 0.24,0 0,12.72 -0.48,0 0,-12.72 0.24,0" /><path
|
||||
inkscape:connector-curvature="0"
|
||||
id="path1806"
|
||||
style="fill:#000000;fill-opacity:1;fill-rule:nonzero;stroke:none"
|
||||
d="m 100.76046,644.95388 0.24,0 0,12.72 -0.6,0 0,-12.72 0.36,0" /><path
|
||||
inkscape:connector-curvature="0"
|
||||
id="path1808"
|
||||
style="fill:#000000;fill-opacity:1;fill-rule:nonzero;stroke:none"
|
||||
d="m 113.36046,644.95388 0.36,0 0,12.72 -0.72,0 0,-12.72 0.36,0" /><path
|
||||
inkscape:connector-curvature="0"
|
||||
id="path1810"
|
||||
style="fill:#000000;fill-opacity:1;fill-rule:nonzero;stroke:none"
|
||||
d="m 138.56046,632.35388 0,0.24 -12.6,0 0,-0.6 12.6,0 0,0.36" /><path
|
||||
inkscape:connector-curvature="0"
|
||||
id="path1812"
|
||||
style="fill:#000000;fill-opacity:1;fill-rule:nonzero;stroke:none"
|
||||
d="m 138.56046,619.63388 0,0.36 -12.6,0 0,-0.72 12.6,0 0,0.36" /><path
|
||||
inkscape:connector-curvature="0"
|
||||
id="path1814"
|
||||
style="fill:#000000;fill-opacity:1;fill-rule:nonzero;stroke:none"
|
||||
d="m 138.56046,607.03388 0,0.24 -12.6,0 0,-0.6 12.6,0 0,0.36" /><path
|
||||
inkscape:connector-curvature="0"
|
||||
id="path1816"
|
||||
style="fill:#000000;fill-opacity:1;fill-rule:nonzero;stroke:none"
|
||||
d="m 138.56046,594.19364 0,0.36 -12.6,0 0,-0.6 12.6,0 0,0.24" /><path
|
||||
inkscape:connector-curvature="0"
|
||||
id="path1818"
|
||||
style="fill:#000000;fill-opacity:1;fill-rule:nonzero;stroke:none"
|
||||
d="m 113.36046,568.87364 0.36,0 0,12.72 -0.72,0 0,-12.72 0.36,0" /><path
|
||||
inkscape:connector-curvature="0"
|
||||
id="path1820"
|
||||
style="fill:#000000;fill-opacity:1;fill-rule:nonzero;stroke:none"
|
||||
d="m 100.76046,568.87364 0.24,0 0,12.72 -0.6,0 0,-12.72 0.36,0" /><path
|
||||
inkscape:connector-curvature="0"
|
||||
id="path1822"
|
||||
style="fill:#000000;fill-opacity:1;fill-rule:nonzero;stroke:none"
|
||||
d="m 88.04046,568.87364 0.24,0 0,12.72 -0.48,0 0,-12.72 0.24,0" /><path
|
||||
inkscape:connector-curvature="0"
|
||||
id="path1824"
|
||||
style="fill:#000000;fill-opacity:1;fill-rule:nonzero;stroke:none"
|
||||
d="m 75.44046,568.87364 0.24,0 0,12.72 -0.6,0 0,-12.72 0.36,0" /><path
|
||||
inkscape:connector-curvature="0"
|
||||
id="path1826"
|
||||
style="fill:#000000;fill-opacity:1;fill-rule:nonzero;stroke:none"
|
||||
d="m 50.12046,594.19364 0,-0.24 12.6,0 0,0.6 -12.6,0 0,-0.36" /><path
|
||||
inkscape:connector-curvature="0"
|
||||
id="path1828"
|
||||
style="fill:#000000;fill-opacity:1;fill-rule:nonzero;stroke:none"
|
||||
d="m 50.12046,607.03388 0,-0.36 12.6,0 0,0.6 -12.6,0 0,-0.24" /><path
|
||||
inkscape:connector-curvature="0"
|
||||
id="path1830"
|
||||
style="fill:#000000;fill-opacity:1;fill-rule:nonzero;stroke:none"
|
||||
d="m 50.12046,619.63388 0,-0.36 12.6,0 0,0.72 -12.6,0 0,-0.36" /><path
|
||||
inkscape:connector-curvature="0"
|
||||
id="path1832"
|
||||
style="fill:#000000;fill-opacity:1;fill-rule:nonzero;stroke:none"
|
||||
d="m 50.12046,632.35388 0,-0.36 12.6,0 0,0.6 -12.6,0 0,-0.24" /></g><flowRoot
|
||||
transform="matrix(0.8,0,0,-0.8,-4.1722236,856.95619)"
|
||||
style="font-style:normal;font-weight:normal;font-size:15px;line-height:125%;font-family:sans-serif;letter-spacing:0px;word-spacing:0px;fill:#000000;fill-opacity:1;stroke:none;stroke-width:1px;stroke-linecap:butt;stroke-linejoin:miter;stroke-opacity:1"
|
||||
id="flowRoot6916"
|
||||
xml:space="preserve"><flowRegion
|
||||
id="flowRegion6918"><rect
|
||||
y="30.103283"
|
||||
x="32.333157"
|
||||
height="313.29712"
|
||||
width="327.79129"
|
||||
id="rect6920" /></flowRegion><flowPara
|
||||
style="font-style:normal;font-variant:normal;font-weight:bold;font-stretch:normal;font-family:sans-serif;-inkscape-font-specification:'sans-serif Bold';text-align:justify;text-anchor:start"
|
||||
id="flowPara6922">Exercice des carrés de Pierre</flowPara><flowPara
|
||||
style="text-align:justify;text-anchor:start"
|
||||
id="flowPara6928" /><flowPara
|
||||
style="text-align:justify;text-anchor:start"
|
||||
id="flowPara6930">Pierre joue avec des mosaïques de couleur. </flowPara><flowPara
|
||||
style="text-align:justify;text-anchor:start"
|
||||
id="flowPara6932">Il dispose ses mosaïques pour obtenir des « carrés »</flowPara><flowPara
|
||||
style="text-align:justify;text-anchor:start"
|
||||
id="flowPara6938" /><flowPara
|
||||
style="text-align:justify;text-anchor:start"
|
||||
id="flowPara6940">Il voudrait savoir à l’avance combien de mosaïques il lui faut pour fabriquer n’importe quel « carré ». </flowPara><flowPara
|
||||
style="text-align:justify;text-anchor:start"
|
||||
id="flowPara6942">Comment l’aider ?</flowPara><flowPara
|
||||
id="flowPara6934" /><flowPara
|
||||
id="flowPara6936" /><flowPara
|
||||
id="flowPara6924" /><flowPara
|
||||
id="flowPara6926" /></flowRoot></g><use
|
||||
x="0"
|
||||
y="0"
|
||||
xlink:href="#g7710"
|
||||
id="use7805"
|
||||
transform="translate(0,400.09945)"
|
||||
width="100%"
|
||||
height="100%" /><use
|
||||
x="0"
|
||||
y="0"
|
||||
xlink:href="#use7805"
|
||||
id="use7807"
|
||||
transform="translate(0,-600.14918)"
|
||||
width="100%"
|
||||
height="100%" /><use
|
||||
x="0"
|
||||
y="0"
|
||||
xlink:href="#use7807"
|
||||
id="use7809"
|
||||
transform="translate(0,400.09945)"
|
||||
width="100%"
|
||||
height="100%" /></g></svg>
|
After Width: | Height: | Size: 31 KiB |
BIN
3e/Expression_litterale/Periode1/carre_pierres_choix_formule.pdf
Normal file
BIN
3e/Expression_litterale/Periode1/carre_pierres_choix_formule.pdf
Normal file
Binary file not shown.
@@ -0,0 +1,67 @@
|
||||
\documentclass[a4paper,12pt, landscape, twocolumn]{/media/documents/Cours/Prof/Enseignements/2016-2017/tools/style/classExo}
|
||||
\usepackage{/media/documents/Cours/Prof/Enseignements/2016-2017/theme}
|
||||
%\geometry{left=10mm,right=10mm, top=10mm, bottom=10mm}
|
||||
|
||||
% Title Page
|
||||
\titre{Trouver la bonne formule}
|
||||
% \seconde \premiereS \PSTMG \TSTMG
|
||||
\classe{Troisième}
|
||||
\date{Septembre 2016}
|
||||
%\duree{1 heure}
|
||||
%\sujet{1}
|
||||
% DS DSCorr DM DMCorr Corr
|
||||
|
||||
%\printanswers
|
||||
\thispagestyle{empty}
|
||||
|
||||
\begin{document}
|
||||
|
||||
\begin{Exo}
|
||||
Pierre dispose maintenant ses mozaïques pour obtenir des carrés à double côté. En voici 2
|
||||
|
||||
\includegraphics[scale=1]{./fig/carre_2eversion}
|
||||
|
||||
Voici plusieurs formules, à toi de trouver la ou les bonnes (dans les formules $c$ désigne le nombre de mozaïque)
|
||||
|
||||
\begin{enumerate}
|
||||
\item $(c-1)\times4 + c - 2$
|
||||
\item $5c -6$
|
||||
\item $4(c-1) + c$
|
||||
\item $c^2 - 12c + 64$
|
||||
\item $c + c - 3 + c + c + c - 3$
|
||||
\item $3 \times c - 2 + 2 \times c - 4$
|
||||
\item $2c + 3(c-2)$
|
||||
\end{enumerate}
|
||||
|
||||
\end{Exo}
|
||||
|
||||
\setcounter{question}{0}
|
||||
\pagebreak
|
||||
|
||||
\begin{Exo}
|
||||
Pierre dispose maintenant ses mozaïques pour obtenir des carrés à double côté. En voici 2
|
||||
|
||||
\includegraphics[scale=1]{./fig/carre_2eversion}
|
||||
|
||||
Voici plusieurs formules, à toi de trouver la ou les bonnes (dans les formules $c$ désigne le nombre de mozaïque)
|
||||
|
||||
\begin{enumerate}
|
||||
\item $(c-1)\times4 + c - 2$
|
||||
\item $5c -6$
|
||||
\item $4(c-1) + c$
|
||||
\item $c^2 - 12c + 64$
|
||||
\item $c + c - 3 + c + c + c - 3$
|
||||
\item $3 \times c - 2 + 2 \times c - 4$
|
||||
\item $2c + 3(c-2)$
|
||||
\end{enumerate}
|
||||
|
||||
\end{Exo}
|
||||
|
||||
\setcounter{question}{0}
|
||||
\pagebreak
|
||||
\end{document}
|
||||
|
||||
%%% Local Variables:
|
||||
%%% mode: latex
|
||||
%%% TeX-master: "master"
|
||||
%%% End:
|
BIN
3e/Expression_litterale/Periode1/fig/carre_2eversion.pdf
Normal file
BIN
3e/Expression_litterale/Periode1/fig/carre_2eversion.pdf
Normal file
Binary file not shown.
370
3e/Expression_litterale/Periode1/fig/carre_2eversion.svg
Normal file
370
3e/Expression_litterale/Periode1/fig/carre_2eversion.svg
Normal file
@@ -0,0 +1,370 @@
|
||||
<?xml version="1.0" encoding="UTF-8" standalone="no"?>
|
||||
<!-- Created with Inkscape (http://www.inkscape.org/) -->
|
||||
|
||||
<svg
|
||||
xmlns:dc="http://purl.org/dc/elements/1.1/"
|
||||
xmlns:cc="http://creativecommons.org/ns#"
|
||||
xmlns:rdf="http://www.w3.org/1999/02/22-rdf-syntax-ns#"
|
||||
xmlns:svg="http://www.w3.org/2000/svg"
|
||||
xmlns="http://www.w3.org/2000/svg"
|
||||
xmlns:sodipodi="http://sodipodi.sourceforge.net/DTD/sodipodi-0.dtd"
|
||||
xmlns:inkscape="http://www.inkscape.org/namespaces/inkscape"
|
||||
id="svg4155"
|
||||
version="1.1"
|
||||
inkscape:version="0.91 r13725"
|
||||
xml:space="preserve"
|
||||
width="316.33911"
|
||||
height="160.95"
|
||||
viewBox="0 0 316.33911 160.95"
|
||||
sodipodi:docname="carre_2eversion.svg"><metadata
|
||||
id="metadata4161"><rdf:RDF><cc:Work
|
||||
rdf:about=""><dc:format>image/svg+xml</dc:format><dc:type
|
||||
rdf:resource="http://purl.org/dc/dcmitype/StillImage" /><dc:title></dc:title></cc:Work></rdf:RDF></metadata><defs
|
||||
id="defs4159"><clipPath
|
||||
clipPathUnits="userSpaceOnUse"
|
||||
id="clipPath4205"><path
|
||||
d="m 4737,389.668 221,0 0,-49 -221,0 0,49 z"
|
||||
id="path4207"
|
||||
inkscape:connector-curvature="0" /></clipPath></defs><sodipodi:namedview
|
||||
pagecolor="#ffffff"
|
||||
bordercolor="#666666"
|
||||
borderopacity="1"
|
||||
objecttolerance="10"
|
||||
gridtolerance="10"
|
||||
guidetolerance="10"
|
||||
inkscape:pageopacity="0"
|
||||
inkscape:pageshadow="2"
|
||||
inkscape:window-width="1366"
|
||||
inkscape:window-height="768"
|
||||
id="namedview4157"
|
||||
showgrid="false"
|
||||
fit-margin-top="0"
|
||||
fit-margin-left="0"
|
||||
fit-margin-right="0"
|
||||
fit-margin-bottom="0"
|
||||
inkscape:zoom="0.74036263"
|
||||
inkscape:cx="282.33948"
|
||||
inkscape:cy="70.891888"
|
||||
inkscape:window-x="0"
|
||||
inkscape:window-y="0"
|
||||
inkscape:window-maximized="1"
|
||||
inkscape:current-layer="g4163" /><g
|
||||
id="g4163"
|
||||
inkscape:groupmode="layer"
|
||||
inkscape:label="exercices12"
|
||||
transform="matrix(1.25,0,0,-1.25,-138.0615,943.69794)"><g
|
||||
id="g4201"
|
||||
transform="scale(0.12,0.12)"><g
|
||||
clip-path="url(#clipPath4205)"
|
||||
id="g4203"><g
|
||||
transform="scale(8.33333,8.33333)"
|
||||
id="g4209" /></g></g><g
|
||||
id="g4219" /><path
|
||||
inkscape:connector-curvature="0"
|
||||
id="path4247"
|
||||
style="fill:#ffffff;fill-opacity:1;fill-rule:nonzero;stroke:none"
|
||||
d="m 640.04479,272.35292 241.56,0 0,204.36 -241.56,0 0,-204.36 z" /><g
|
||||
id="g4832"
|
||||
transform="translate(-512.44429,232.76543)"><path
|
||||
d="m 747.68479,521.83292 0,-0.36 127.92,0 0,0.72 -127.92,0 0,-0.36"
|
||||
style="fill:#000000;fill-opacity:1;fill-rule:nonzero;stroke:none"
|
||||
id="path4251"
|
||||
inkscape:connector-curvature="0" /><path
|
||||
d="m 747.68479,393.79292 0.36,0 0,128.04 -0.6,0 0,-128.04 0.24,0"
|
||||
style="fill:#000000;fill-opacity:1;fill-rule:nonzero;stroke:none"
|
||||
id="path4253"
|
||||
inkscape:connector-curvature="0" /><path
|
||||
d="m 875.60479,393.79292 0,0.36 -127.92,0 0,-0.72 127.92,0 0,0.36"
|
||||
style="fill:#000000;fill-opacity:1;fill-rule:nonzero;stroke:none"
|
||||
id="path4255"
|
||||
inkscape:connector-curvature="0" /><path
|
||||
d="m 875.60479,521.83292 -0.36,0 0,-128.04 0.72,0 0,128.04 -0.36,0"
|
||||
style="fill:#000000;fill-opacity:1;fill-rule:nonzero;stroke:none"
|
||||
id="path4257"
|
||||
inkscape:connector-curvature="0" /><path
|
||||
d="m 875.60479,508.99292 0,0.36 -127.92,0 0,-0.6 127.92,0 0,0.24"
|
||||
style="fill:#000000;fill-opacity:1;fill-rule:nonzero;stroke:none"
|
||||
id="path4259"
|
||||
inkscape:connector-curvature="0" /><path
|
||||
d="m 862.76479,393.79292 0.24,0 0,128.04 -0.6,0 0,-128.04 0.36,0"
|
||||
style="fill:#000000;fill-opacity:1;fill-rule:nonzero;stroke:none"
|
||||
id="path4261"
|
||||
inkscape:connector-curvature="0" /><path
|
||||
d="m 875.60479,406.63292 0,0.24 -127.92,0 0,-0.6 127.92,0 0,0.36"
|
||||
style="fill:#000000;fill-opacity:1;fill-rule:nonzero;stroke:none"
|
||||
id="path4263"
|
||||
inkscape:connector-curvature="0" /><path
|
||||
d="m 760.52479,393.79292 0.24,0 0,128.04 -0.6,0 0,-128.04 0.36,0"
|
||||
style="fill:#000000;fill-opacity:1;fill-rule:nonzero;stroke:none"
|
||||
id="path4265"
|
||||
inkscape:connector-curvature="0" /><path
|
||||
d="m 760.52479,496.27292 0,0.36 -12.84,0 0,-0.72 12.84,0 0,0.36"
|
||||
style="fill:#000000;fill-opacity:1;fill-rule:nonzero;stroke:none"
|
||||
id="path4267"
|
||||
inkscape:connector-curvature="0" /><path
|
||||
d="m 760.52479,483.43292 0,0.36 -12.84,0 0,-0.6 12.84,0 0,0.24"
|
||||
style="fill:#000000;fill-opacity:1;fill-rule:nonzero;stroke:none"
|
||||
id="path4269"
|
||||
inkscape:connector-curvature="0" /><path
|
||||
d="m 760.52479,470.71292 0,0.24 -12.84,0 0,-0.6 12.84,0 0,0.36"
|
||||
style="fill:#000000;fill-opacity:1;fill-rule:nonzero;stroke:none"
|
||||
id="path4271"
|
||||
inkscape:connector-curvature="0" /><path
|
||||
d="m 760.52479,457.87292 0,0.24 -12.84,0 0,-0.6 12.84,0 0,0.36"
|
||||
style="fill:#000000;fill-opacity:1;fill-rule:nonzero;stroke:none"
|
||||
id="path4273"
|
||||
inkscape:connector-curvature="0" /><path
|
||||
d="m 760.52479,445.03292 0,0.24 -12.84,0 0,-0.6 12.84,0 0,0.36"
|
||||
style="fill:#000000;fill-opacity:1;fill-rule:nonzero;stroke:none"
|
||||
id="path4275"
|
||||
inkscape:connector-curvature="0" /><path
|
||||
d="m 760.52479,432.19292 0,0.36 -12.84,0 0,-0.72 12.84,0 0,0.36"
|
||||
style="fill:#000000;fill-opacity:1;fill-rule:nonzero;stroke:none"
|
||||
id="path4277"
|
||||
inkscape:connector-curvature="0" /><path
|
||||
d="m 760.52479,419.35292 0,0.36 -12.84,0 0,-0.6 12.84,0 0,0.24"
|
||||
style="fill:#000000;fill-opacity:1;fill-rule:nonzero;stroke:none"
|
||||
id="path4279"
|
||||
inkscape:connector-curvature="0" /><path
|
||||
d="m 773.24479,508.99292 0.36,0 0,12.84 -0.72,0 0,-12.84 0.36,0"
|
||||
style="fill:#000000;fill-opacity:1;fill-rule:nonzero;stroke:none"
|
||||
id="path4281"
|
||||
inkscape:connector-curvature="0" /><path
|
||||
d="m 786.08479,508.99292 0.24,0 0,12.84 -0.6,0 0,-12.84 0.36,0"
|
||||
style="fill:#000000;fill-opacity:1;fill-rule:nonzero;stroke:none"
|
||||
id="path4283"
|
||||
inkscape:connector-curvature="0" /><path
|
||||
d="m 798.80479,508.99292 0.36,0 0,12.84 -0.72,0 0,-12.84 0.36,0"
|
||||
style="fill:#000000;fill-opacity:1;fill-rule:nonzero;stroke:none"
|
||||
id="path4285"
|
||||
inkscape:connector-curvature="0" /><path
|
||||
d="m 811.64479,508.99292 0.36,0 0,12.84 -0.72,0 0,-12.84 0.36,0"
|
||||
style="fill:#000000;fill-opacity:1;fill-rule:nonzero;stroke:none"
|
||||
id="path4287"
|
||||
inkscape:connector-curvature="0" /><path
|
||||
d="m 824.48479,508.99292 0.24,0 0,12.84 -0.6,0 0,-12.84 0.36,0"
|
||||
style="fill:#000000;fill-opacity:1;fill-rule:nonzero;stroke:none"
|
||||
id="path4289"
|
||||
inkscape:connector-curvature="0" /><path
|
||||
d="m 837.20479,508.99292 0.36,0 0,12.84 -0.72,0 0,-12.84 0.36,0"
|
||||
style="fill:#000000;fill-opacity:1;fill-rule:nonzero;stroke:none"
|
||||
id="path4291"
|
||||
inkscape:connector-curvature="0" /><path
|
||||
d="m 849.92479,508.99292 0.36,0 0,12.84 -0.6,0 0,-12.84 0.24,0"
|
||||
style="fill:#000000;fill-opacity:1;fill-rule:nonzero;stroke:none"
|
||||
id="path4293"
|
||||
inkscape:connector-curvature="0" /><path
|
||||
d="m 875.60479,496.27292 0,0.36 -12.84,0 0,-0.72 12.84,0 0,0.36"
|
||||
style="fill:#000000;fill-opacity:1;fill-rule:nonzero;stroke:none"
|
||||
id="path4295"
|
||||
inkscape:connector-curvature="0" /><path
|
||||
d="m 875.60479,483.43292 0,0.36 -12.84,0 0,-0.6 12.84,0 0,0.24"
|
||||
style="fill:#000000;fill-opacity:1;fill-rule:nonzero;stroke:none"
|
||||
id="path4297"
|
||||
inkscape:connector-curvature="0" /><path
|
||||
d="m 875.60479,470.71292 0,0.24 -12.84,0 0,-0.6 12.84,0 0,0.36"
|
||||
style="fill:#000000;fill-opacity:1;fill-rule:nonzero;stroke:none"
|
||||
id="path4299"
|
||||
inkscape:connector-curvature="0" /><path
|
||||
d="m 875.60479,457.87292 0,0.24 -12.84,0 0,-0.6 12.84,0 0,0.36"
|
||||
style="fill:#000000;fill-opacity:1;fill-rule:nonzero;stroke:none"
|
||||
id="path4301"
|
||||
inkscape:connector-curvature="0" /><path
|
||||
d="m 875.60479,445.03292 0,0.24 -12.84,0 0,-0.6 12.84,0 0,0.36"
|
||||
style="fill:#000000;fill-opacity:1;fill-rule:nonzero;stroke:none"
|
||||
id="path4303"
|
||||
inkscape:connector-curvature="0" /><path
|
||||
d="m 875.60479,432.19292 0,0.36 -12.84,0 0,-0.72 12.84,0 0,0.36"
|
||||
style="fill:#000000;fill-opacity:1;fill-rule:nonzero;stroke:none"
|
||||
id="path4305"
|
||||
inkscape:connector-curvature="0" /><path
|
||||
d="m 875.60479,419.35292 0,0.36 -12.84,0 0,-0.6 12.84,0 0,0.24"
|
||||
style="fill:#000000;fill-opacity:1;fill-rule:nonzero;stroke:none"
|
||||
id="path4307"
|
||||
inkscape:connector-curvature="0" /><path
|
||||
d="m 849.92479,393.79292 0.36,0 0,12.84 -0.6,0 0,-12.84 0.24,0"
|
||||
style="fill:#000000;fill-opacity:1;fill-rule:nonzero;stroke:none"
|
||||
id="path4309"
|
||||
inkscape:connector-curvature="0" /><path
|
||||
d="m 837.20479,393.79292 0.36,0 0,12.84 -0.72,0 0,-12.84 0.36,0"
|
||||
style="fill:#000000;fill-opacity:1;fill-rule:nonzero;stroke:none"
|
||||
id="path4311"
|
||||
inkscape:connector-curvature="0" /><path
|
||||
d="m 824.48479,393.79292 0.24,0 0,12.84 -0.6,0 0,-12.84 0.36,0"
|
||||
style="fill:#000000;fill-opacity:1;fill-rule:nonzero;stroke:none"
|
||||
id="path4313"
|
||||
inkscape:connector-curvature="0" /><path
|
||||
d="m 811.64479,393.79292 0.36,0 0,12.84 -0.72,0 0,-12.84 0.36,0"
|
||||
style="fill:#000000;fill-opacity:1;fill-rule:nonzero;stroke:none"
|
||||
id="path4315"
|
||||
inkscape:connector-curvature="0" /><path
|
||||
d="m 798.80479,393.79292 0.36,0 0,12.84 -0.72,0 0,-12.84 0.36,0"
|
||||
style="fill:#000000;fill-opacity:1;fill-rule:nonzero;stroke:none"
|
||||
id="path4317"
|
||||
inkscape:connector-curvature="0" /><path
|
||||
d="m 786.08479,393.79292 0.24,0 0,12.84 -0.6,0 0,-12.84 0.36,0"
|
||||
style="fill:#000000;fill-opacity:1;fill-rule:nonzero;stroke:none"
|
||||
id="path4319"
|
||||
inkscape:connector-curvature="0" /><path
|
||||
d="m 773.24479,393.79292 0.36,0 0,12.84 -0.72,0 0,-12.84 0.36,0"
|
||||
style="fill:#000000;fill-opacity:1;fill-rule:nonzero;stroke:none"
|
||||
id="path4321"
|
||||
inkscape:connector-curvature="0" /><path
|
||||
d="m 773.24479,496.27292 0.36,0 0,12.72 -0.72,0 0,-12.72 0.36,0"
|
||||
style="fill:#000000;fill-opacity:1;fill-rule:nonzero;stroke:none"
|
||||
id="path4391"
|
||||
inkscape:connector-curvature="0" /><path
|
||||
d="m 773.24479,406.63292 0.36,0 0,12.72 -0.72,0 0,-12.72 0.36,0"
|
||||
style="fill:#000000;fill-opacity:1;fill-rule:nonzero;stroke:none"
|
||||
id="path4393"
|
||||
inkscape:connector-curvature="0" /><path
|
||||
d="m 773.24479,419.35292 0.36,0 0,76.92 -0.72,0 0,-76.92 0.36,0"
|
||||
style="fill:#000000;fill-opacity:1;fill-rule:nonzero;stroke:none"
|
||||
id="path4395"
|
||||
inkscape:connector-curvature="0" /><path
|
||||
d="m 773.24479,483.43292 0,0.36 -12.72,0 0,-0.6 12.72,0 0,0.24"
|
||||
style="fill:#000000;fill-opacity:1;fill-rule:nonzero;stroke:none"
|
||||
id="path4397"
|
||||
inkscape:connector-curvature="0" /><path
|
||||
d="m 773.24479,470.71292 0,0.24 -12.72,0 0,-0.6 12.72,0 0,0.36"
|
||||
style="fill:#000000;fill-opacity:1;fill-rule:nonzero;stroke:none"
|
||||
id="path4399"
|
||||
inkscape:connector-curvature="0" /><path
|
||||
d="m 773.24479,457.87292 0,0.24 -12.72,0 0,-0.6 12.72,0 0,0.36"
|
||||
style="fill:#000000;fill-opacity:1;fill-rule:nonzero;stroke:none"
|
||||
id="path4401"
|
||||
inkscape:connector-curvature="0" /><path
|
||||
d="m 773.24479,445.03292 0,0.24 -12.72,0 0,-0.6 12.72,0 0,0.36"
|
||||
style="fill:#000000;fill-opacity:1;fill-rule:nonzero;stroke:none"
|
||||
id="path4403"
|
||||
inkscape:connector-curvature="0" /><path
|
||||
d="m 773.24479,432.19292 0,0.36 -12.72,0 0,-0.72 12.72,0 0,0.36"
|
||||
style="fill:#000000;fill-opacity:1;fill-rule:nonzero;stroke:none"
|
||||
id="path4405"
|
||||
inkscape:connector-curvature="0" /><path
|
||||
d="m 773.24479,496.27292 0,0.36 -12.72,0 0,-0.72 12.72,0 0,0.36"
|
||||
style="fill:#000000;fill-opacity:1;fill-rule:nonzero;stroke:none"
|
||||
id="path4431"
|
||||
inkscape:connector-curvature="0" /><path
|
||||
d="m 773.24479,419.35292 0,0.36 -12.72,0 0,-0.6 12.72,0 0,0.24"
|
||||
style="fill:#000000;fill-opacity:1;fill-rule:nonzero;stroke:none"
|
||||
id="path4433"
|
||||
inkscape:connector-curvature="0" /></g><g
|
||||
id="g4801"
|
||||
transform="translate(-534.63559,316.04543)"><path
|
||||
d="m 734.84479,419.35292 0,0.36 -89.4,0 0,-0.6 89.4,0 0,0.24"
|
||||
style="fill:#000000;fill-opacity:1;fill-rule:nonzero;stroke:none"
|
||||
id="path4323"
|
||||
inkscape:connector-curvature="0" /><path
|
||||
d="m 734.84479,329.71292 0.36,0 0,89.64 -0.72,0 0,-89.64 0.36,0"
|
||||
style="fill:#000000;fill-opacity:1;fill-rule:nonzero;stroke:none"
|
||||
id="path4325"
|
||||
inkscape:connector-curvature="0" /><path
|
||||
d="m 645.44479,329.71292 0,-0.36 89.4,0 0,0.72 -89.4,0 0,-0.36"
|
||||
style="fill:#000000;fill-opacity:1;fill-rule:nonzero;stroke:none"
|
||||
id="path4327"
|
||||
inkscape:connector-curvature="0" /><path
|
||||
d="m 645.44479,419.35292 -0.36,0 0,-89.64 0.6,0 0,89.64 -0.24,0"
|
||||
style="fill:#000000;fill-opacity:1;fill-rule:nonzero;stroke:none"
|
||||
id="path4329"
|
||||
inkscape:connector-curvature="0" /><path
|
||||
d="m 734.84479,406.63292 0,0.24 -89.4,0 0,-0.6 89.4,0 0,0.36"
|
||||
style="fill:#000000;fill-opacity:1;fill-rule:nonzero;stroke:none"
|
||||
id="path4331"
|
||||
inkscape:connector-curvature="0" /><path
|
||||
d="m 722.12479,329.71292 0.24,0 0,89.64 -0.6,0 0,-89.64 0.36,0"
|
||||
style="fill:#000000;fill-opacity:1;fill-rule:nonzero;stroke:none"
|
||||
id="path4333"
|
||||
inkscape:connector-curvature="0" /><path
|
||||
d="m 645.44479,342.55292 0,-0.36 89.4,0 0,0.6 -89.4,0 0,-0.24"
|
||||
style="fill:#000000;fill-opacity:1;fill-rule:nonzero;stroke:none"
|
||||
id="path4335"
|
||||
inkscape:connector-curvature="0" /><path
|
||||
d="m 658.16479,329.71292 0.36,0 0,89.64 -0.72,0 0,-89.64 0.36,0"
|
||||
style="fill:#000000;fill-opacity:1;fill-rule:nonzero;stroke:none"
|
||||
id="path4337"
|
||||
inkscape:connector-curvature="0" /><path
|
||||
d="m 670.88479,406.63292 0.36,0 0,12.72 -0.6,0 0,-12.72 0.24,0"
|
||||
style="fill:#000000;fill-opacity:1;fill-rule:nonzero;stroke:none"
|
||||
id="path4339"
|
||||
inkscape:connector-curvature="0" /><path
|
||||
d="m 683.84479,406.63292 0.24,0 0,12.72 -0.6,0 0,-12.72 0.36,0"
|
||||
style="fill:#000000;fill-opacity:1;fill-rule:nonzero;stroke:none"
|
||||
id="path4341"
|
||||
inkscape:connector-curvature="0" /><path
|
||||
d="m 696.56479,406.63292 0.36,0 0,12.72 -0.72,0 0,-12.72 0.36,0"
|
||||
style="fill:#000000;fill-opacity:1;fill-rule:nonzero;stroke:none"
|
||||
id="path4343"
|
||||
inkscape:connector-curvature="0" /><path
|
||||
d="m 709.28479,406.63292 0.36,0 0,12.72 -0.6,0 0,-12.72 0.24,0"
|
||||
style="fill:#000000;fill-opacity:1;fill-rule:nonzero;stroke:none"
|
||||
id="path4345"
|
||||
inkscape:connector-curvature="0" /><path
|
||||
d="m 734.84479,393.79292 0,0.36 -12.72,0 0,-0.72 12.72,0 0,0.36"
|
||||
style="fill:#000000;fill-opacity:1;fill-rule:nonzero;stroke:none"
|
||||
id="path4347"
|
||||
inkscape:connector-curvature="0" /><path
|
||||
d="m 734.84479,380.95292 0,0.36 -12.72,0 0,-0.6 12.72,0 0,0.24"
|
||||
style="fill:#000000;fill-opacity:1;fill-rule:nonzero;stroke:none"
|
||||
id="path4349"
|
||||
inkscape:connector-curvature="0" /><path
|
||||
d="m 734.84479,368.11292 0,0.36 -12.72,0 0,-0.6 12.72,0 0,0.24"
|
||||
style="fill:#000000;fill-opacity:1;fill-rule:nonzero;stroke:none"
|
||||
id="path4351"
|
||||
inkscape:connector-curvature="0" /><path
|
||||
d="m 734.84479,355.27292 0,0.36 -12.72,0 0,-0.6 12.72,0 0,0.24"
|
||||
style="fill:#000000;fill-opacity:1;fill-rule:nonzero;stroke:none"
|
||||
id="path4353"
|
||||
inkscape:connector-curvature="0" /><path
|
||||
d="m 709.28479,329.71292 0.36,0 0,12.84 -0.6,0 0,-12.84 0.24,0"
|
||||
style="fill:#000000;fill-opacity:1;fill-rule:nonzero;stroke:none"
|
||||
id="path4355"
|
||||
inkscape:connector-curvature="0" /><path
|
||||
d="m 696.56479,329.71292 0.36,0 0,12.84 -0.72,0 0,-12.84 0.36,0"
|
||||
style="fill:#000000;fill-opacity:1;fill-rule:nonzero;stroke:none"
|
||||
id="path4357"
|
||||
inkscape:connector-curvature="0" /><path
|
||||
d="m 683.84479,329.71292 0.24,0 0,12.84 -0.6,0 0,-12.84 0.36,0"
|
||||
style="fill:#000000;fill-opacity:1;fill-rule:nonzero;stroke:none"
|
||||
id="path4359"
|
||||
inkscape:connector-curvature="0" /><path
|
||||
d="m 670.88479,329.71292 0.36,0 0,12.84 -0.6,0 0,-12.84 0.24,0"
|
||||
style="fill:#000000;fill-opacity:1;fill-rule:nonzero;stroke:none"
|
||||
id="path4361"
|
||||
inkscape:connector-curvature="0" /><path
|
||||
d="m 645.44479,355.27292 0,-0.24 12.72,0 0,0.6 -12.72,0 0,-0.36"
|
||||
style="fill:#000000;fill-opacity:1;fill-rule:nonzero;stroke:none"
|
||||
id="path4363"
|
||||
inkscape:connector-curvature="0" /><path
|
||||
d="m 645.44479,368.11292 0,-0.24 12.72,0 0,0.6 -12.72,0 0,-0.36"
|
||||
style="fill:#000000;fill-opacity:1;fill-rule:nonzero;stroke:none"
|
||||
id="path4365"
|
||||
inkscape:connector-curvature="0" /><path
|
||||
d="m 645.44479,380.95292 0,-0.24 12.72,0 0,0.6 -12.72,0 0,-0.36"
|
||||
style="fill:#000000;fill-opacity:1;fill-rule:nonzero;stroke:none"
|
||||
id="path4367"
|
||||
inkscape:connector-curvature="0" /><path
|
||||
d="m 645.44479,393.79292 0,-0.36 12.72,0 0,0.72 -12.72,0 0,-0.36"
|
||||
style="fill:#000000;fill-opacity:1;fill-rule:nonzero;stroke:none"
|
||||
id="path4369"
|
||||
inkscape:connector-curvature="0" /><path
|
||||
d="m 670.88479,342.55292 0.36,0 0,64.08 -0.6,0 0,-64.08 0.24,0"
|
||||
style="fill:#000000;fill-opacity:1;fill-rule:nonzero;stroke:none"
|
||||
id="path4407"
|
||||
inkscape:connector-curvature="0" /><path
|
||||
d="m 670.88479,380.95292 0,0.36 -12.72,0 0,-0.6 12.72,0 0,0.24"
|
||||
style="fill:#000000;fill-opacity:1;fill-rule:nonzero;stroke:none"
|
||||
id="path4409"
|
||||
inkscape:connector-curvature="0" /><path
|
||||
d="m 670.88479,368.11292 0,0.36 -12.72,0 0,-0.6 12.72,0 0,0.24"
|
||||
style="fill:#000000;fill-opacity:1;fill-rule:nonzero;stroke:none"
|
||||
id="path4411"
|
||||
inkscape:connector-curvature="0" /><path
|
||||
d="m 670.88479,393.79292 0,0.36 -12.72,0 0,-0.72 12.72,0 0,0.36"
|
||||
style="fill:#000000;fill-opacity:1;fill-rule:nonzero;stroke:none"
|
||||
id="path4435"
|
||||
inkscape:connector-curvature="0" /><path
|
||||
d="m 670.88479,355.27292 0,0.36 -12.72,0 0,-0.6 12.72,0 0,0.24"
|
||||
style="fill:#000000;fill-opacity:1;fill-rule:nonzero;stroke:none"
|
||||
id="path4437"
|
||||
inkscape:connector-curvature="0" /></g></g></svg>
|
After Width: | Height: | Size: 19 KiB |
BIN
3e/Expression_litterale/Periode1/formule_double_carre.ods
Normal file
BIN
3e/Expression_litterale/Periode1/formule_double_carre.ods
Normal file
Binary file not shown.
37
3e/Expression_litterale/Periode1/index.rst
Normal file
37
3e/Expression_litterale/Periode1/index.rst
Normal file
@@ -0,0 +1,37 @@
|
||||
Notes autour de la 3e période sur le calcul littéral pour les 3e: modélisation
|
||||
##############################################################################
|
||||
|
||||
:date: 2016-08-11
|
||||
:modified: 2016-08-11
|
||||
:tags: Calcul litteral
|
||||
:category: 3e
|
||||
:authors: Bertrand Benjamin
|
||||
:summary: Organisation de l'année pour le calcul littéral pour les 3e
|
||||
|
||||
|
||||
Séance 1 et 2
|
||||
-------------
|
||||
|
||||
Activité sur les `carrés de pierres <./carre_pierres.pdf>`_.
|
||||
|
||||
La première séance sert à la construction de formules.
|
||||
|
||||
La deuxième séance sera utilisée pour apprendre à manipuler ces formules:
|
||||
|
||||
- Calculer le nombre de mosaïques nécessaires
|
||||
- Taille des carrés de pierres max avec un nombre de mosaïques donné
|
||||
- Se demander si d'autres formules sont bonnes.
|
||||
|
||||
Séance 3
|
||||
--------
|
||||
|
||||
`Nouvelle structure de carré de pierres <./carre_pierres_choix_formule.pdf>`_ avec formules à valider.
|
||||
L'objectif étant d'évaluer des formules.
|
||||
|
||||
Séance 4 et suivantes
|
||||
---------------------
|
||||
|
||||
`Exercices techniques <./all_entrainement_eval.pdf>`_ à faire sur la durée.
|
||||
|
||||
C'est aussi l'occasion de dédier une séance à l'initiation au QCM (`initiation au QCM <./QCM.pdf>`_).
|
||||
|
177
3e/Expression_litterale/Periode1/tpl_entrainement_eval.tex
Normal file
177
3e/Expression_litterale/Periode1/tpl_entrainement_eval.tex
Normal file
@@ -0,0 +1,177 @@
|
||||
\documentclass[a4paper,10pt]{/media/documents/Cours/Prof/Enseignements/2016-2017/tools/style/classExo}
|
||||
\usepackage{/media/documents/Cours/Prof/Enseignements/2016-2017/theme}
|
||||
%\geometry{left=10mm,right=10mm, top=10mm, bottom=10mm}
|
||||
|
||||
% Title Page
|
||||
\titre{Technique: calcul et évaluation}
|
||||
% \seconde \premiereS \PSTMG \TSTMG
|
||||
\classe{Troisième}
|
||||
\date{Septembre 2016}
|
||||
%\duree{1 heure}
|
||||
\sujet{\Var{infos.num}}
|
||||
% DS DSCorr DM DMCorr Corr
|
||||
|
||||
%\printanswers
|
||||
\pagestyle{empty}
|
||||
|
||||
\newcommand{\lesexos}{%
|
||||
\begin{Exo}
|
||||
Faire les calculs suivants en détaillant des étapes.
|
||||
\begin{multicols}{3}
|
||||
\begin{enumerate}
|
||||
\Block{set e = Expression.random("{a} + {b}*{c}")}
|
||||
\item $A = \Var{e}$
|
||||
\begin{solution}
|
||||
\begin{eqnarray*}
|
||||
\Var{e.simplify().explain() | calculus(name = "A")}
|
||||
\end{eqnarray*}
|
||||
\end{solution}
|
||||
\Block{set e = Expression.random("{a} - {b}*{c}")}
|
||||
\item $B = \Var{e}$
|
||||
\begin{solution}
|
||||
\begin{eqnarray*}
|
||||
\Var{e.simplify().explain() | calculus(name = "A")}
|
||||
\end{eqnarray*}
|
||||
\end{solution}
|
||||
\Block{set e = Expression.random("{a} * {b} + {c} * {d}")}
|
||||
\item $C = \Var{e}$
|
||||
\begin{solution}
|
||||
\begin{eqnarray*}
|
||||
\Var{e.simplify().explain() | calculus(name = "A")}
|
||||
\end{eqnarray*}
|
||||
\end{solution}
|
||||
\columnbreak
|
||||
\Block{set e = Expression.random("({a} + {b})*{c} + {d}")}
|
||||
\item $D = \Var{e}$
|
||||
\begin{solution}
|
||||
\begin{eqnarray*}
|
||||
\Var{e.simplify().explain() | calculus(name = "A")}
|
||||
\end{eqnarray*}
|
||||
\end{solution}
|
||||
\Block{set e = Expression.random("{a} * ({b} + {c}) * {d}")}
|
||||
\item $E = \Var{e}$
|
||||
\begin{solution}
|
||||
\begin{eqnarray*}
|
||||
\Var{e.simplify().explain() | calculus(name = "A")}
|
||||
\end{eqnarray*}
|
||||
\end{solution}
|
||||
\Block{set e = Expression.random("({a} + {b})({c} - {d})")}
|
||||
\item $F = \Var{e}$
|
||||
\begin{solution}
|
||||
\begin{eqnarray*}
|
||||
\Var{e.simplify().explain() | calculus(name = "F")}
|
||||
\end{eqnarray*}
|
||||
\end{solution}
|
||||
\columnbreak
|
||||
\Block{set e = Expression.random("{a}({b}+{c}) + {d}")}
|
||||
\item $G = \Var{e}$
|
||||
\begin{solution}
|
||||
\begin{eqnarray*}
|
||||
\Var{e.simplify().explain() | calculus(name = "G")}
|
||||
\end{eqnarray*}
|
||||
\end{solution}
|
||||
\Block{set e = Expression.random("{a} - ({b} + {c}) * {d}")}
|
||||
\item $H = \Var{e}$
|
||||
\begin{solution}
|
||||
\begin{eqnarray*}
|
||||
\Var{e.simplify().explain() | calculus(name = "H")}
|
||||
\end{eqnarray*}
|
||||
\end{solution}
|
||||
\Block{set e = Expression.random("{c}({a} - {b} * {d})")}
|
||||
\item $I = \Var{e}$
|
||||
\begin{solution}
|
||||
\begin{eqnarray*}
|
||||
\Var{e.simplify().explain() | calculus(name = "I")}
|
||||
\end{eqnarray*}
|
||||
\end{solution}
|
||||
\end{enumerate}
|
||||
\end{multicols}
|
||||
\end{Exo}
|
||||
|
||||
\begin{Exo}
|
||||
Évaluer les expressions suivantes
|
||||
|
||||
\begin{multicols}{3}
|
||||
\begin{enumerate}
|
||||
\Block{set P = Polynom.random(degree = 1)}
|
||||
\Block{set x = Expression.random("{a}")}
|
||||
\item $\Var{P}$ en $x = \Var{x}$
|
||||
\begin{solution}
|
||||
\begin{eqnarray*}
|
||||
\Var{P(x).explain()| calculus()}
|
||||
\end{eqnarray*}
|
||||
\end{solution}
|
||||
\Block{set P = Polynom.random(degree = 1)}
|
||||
\Block{set x = Expression.random("{a}")}
|
||||
\item $\Var{P}$ en $x = \Var{x}$
|
||||
\begin{solution}
|
||||
\begin{eqnarray*}
|
||||
\Var{P(x).explain()| calculus()}
|
||||
\end{eqnarray*}
|
||||
\end{solution}
|
||||
\Block{set x = Expression.random("{a}")}
|
||||
\item $\Var{P}$ en $x = \Var{x}$
|
||||
\begin{solution}
|
||||
\begin{eqnarray*}
|
||||
\Var{P(x).explain()| calculus()}
|
||||
\end{eqnarray*}
|
||||
\end{solution}
|
||||
|
||||
\Block{set P = Polynom.random(degree = 2)}
|
||||
\Block{set x = Expression.random("{a}")}
|
||||
\item $\Var{P}$ en $x = \Var{x}$
|
||||
\begin{solution}
|
||||
\begin{eqnarray*}
|
||||
\Var{P(x).explain()| calculus()}
|
||||
\end{eqnarray*}
|
||||
\end{solution}
|
||||
\Block{set x = Expression.random("{a}")}
|
||||
\item $\Var{P}$ en $x = \Var{x}$
|
||||
\begin{solution}
|
||||
\begin{eqnarray*}
|
||||
\Var{P(x).explain()| calculus()}
|
||||
\end{eqnarray*}
|
||||
\end{solution}
|
||||
\Block{set P = Polynom.random(degree = 2)}
|
||||
\Block{set x = Expression.random("{a}")}
|
||||
\item $\Var{P}$ en $x = \Var{x}$
|
||||
\begin{solution}
|
||||
\begin{eqnarray*}
|
||||
\Var{P(x).explain()| calculus()}
|
||||
\end{eqnarray*}
|
||||
\end{solution}
|
||||
\Block{set P = Expression.random("{a} x * (x - {b})")}
|
||||
\Block{set x = Expression.random("{a}")}
|
||||
\item $\Var{P}$ en $x = \Var{x}$
|
||||
\Block{set x = Expression.random("{a}")}
|
||||
\item $\Var{P}$ en $x = \Var{x}$
|
||||
\Block{set P = Expression.random("({a}x + {b})({c} + {d}x)")}
|
||||
\Block{set x = Expression.random("{a}")}
|
||||
\item $\Var{P}$ en $x = \Var{x}$
|
||||
\end{enumerate}
|
||||
\end{multicols}
|
||||
\end{Exo}
|
||||
}
|
||||
|
||||
\begin{document}
|
||||
|
||||
\lesexos
|
||||
\setcounter{exo}{0}
|
||||
\vfill
|
||||
|
||||
\lesexos
|
||||
\setcounter{exo}{0}
|
||||
\vfill
|
||||
|
||||
|
||||
\lesexos
|
||||
\setcounter{exo}{0}
|
||||
\vfill
|
||||
|
||||
\end{document}
|
||||
|
||||
%%% Local Variables:
|
||||
%%% mode: latex
|
||||
%%% TeX-master: "master"
|
||||
%%% End:
|
||||
|
240
3e/Expression_litterale/Periode2/01_eval_red_tech.tex
Normal file
240
3e/Expression_litterale/Periode2/01_eval_red_tech.tex
Normal file
@@ -0,0 +1,240 @@
|
||||
\documentclass[a4paper,10pt]{/media/documents/Cours/Prof/Enseignements/2016-2017/tools/style/classExo}
|
||||
\usepackage{/media/documents/Cours/Prof/Enseignements/2016-2017/theme}
|
||||
%\geometry{left=10mm,right=10mm, top=10mm, bottom=10mm}
|
||||
|
||||
% Title Page
|
||||
\titre{Technique: Evaluer et reduire}
|
||||
% \seconde \premiereS \PSTMG \TSTMG
|
||||
\classe{Troisième}
|
||||
\date{Novembre 2016}
|
||||
%\duree{1 heure}
|
||||
\sujet{01}
|
||||
% DS DSCorr DM DMCorr Corr
|
||||
|
||||
%\printanswers
|
||||
\pagestyle{empty}
|
||||
|
||||
\newcommand{\lesexos}{%
|
||||
\begin{Exo}
|
||||
Évaluer les expressions suivantes
|
||||
|
||||
\begin{multicols}{3}
|
||||
\begin{enumerate}
|
||||
|
||||
|
||||
\item $- x - 5$ en $x = 6$
|
||||
\begin{solution}
|
||||
\begin{eqnarray*}
|
||||
A & = & - 6 - 5 \\
|
||||
A & = & -6 - 5 \\
|
||||
A & = & -11
|
||||
\end{eqnarray*}
|
||||
\end{solution}
|
||||
|
||||
|
||||
\item $- x + 8$ en $x = 1$
|
||||
\begin{solution}
|
||||
\begin{eqnarray*}
|
||||
A & = & - 1 + 8 \\
|
||||
A & = & -1 + 8 \\
|
||||
A & = & 7
|
||||
\end{eqnarray*}
|
||||
\end{solution}
|
||||
|
||||
\item $- x + 8$ en $x = 3$
|
||||
\begin{solution}
|
||||
\begin{eqnarray*}
|
||||
A & = & - 3 + 8 \\
|
||||
A & = & -3 + 8 \\
|
||||
A & = & 5
|
||||
\end{eqnarray*}
|
||||
\end{solution}
|
||||
|
||||
|
||||
|
||||
\item $-10 x^{ 2 } + x - 6$ en $x = 7$
|
||||
\begin{solution}
|
||||
\begin{eqnarray*}
|
||||
A & = & -10 \times 7^{ 2 } + 7 - 6 \\
|
||||
A & = & -10 \times 49 + 7 - 6 \\
|
||||
A & = & -490 + 7 - 6 \\
|
||||
A & = & -483 - 6 \\
|
||||
A & = & -489
|
||||
\end{eqnarray*}
|
||||
\end{solution}
|
||||
|
||||
\item $-10 x^{ 2 } + x - 6$ en $x = -5$
|
||||
\begin{solution}
|
||||
\begin{eqnarray*}
|
||||
A & = & -10 \times ( -5 )^{ 2 } - 5 - 6 \\
|
||||
A & = & -10 \times 25 - 5 - 6 \\
|
||||
A & = & -250 - 5 - 6 \\
|
||||
A & = & -255 - 6 \\
|
||||
A & = & -261
|
||||
\end{eqnarray*}
|
||||
\end{solution}
|
||||
|
||||
|
||||
\item $-10 x^{ 2 } + x + 10$ en $x = 8$
|
||||
\begin{solution}
|
||||
\begin{eqnarray*}
|
||||
A & = & -10 \times 8^{ 2 } + 8 + 10 \\
|
||||
A & = & -10 \times 64 + 8 + 10 \\
|
||||
A & = & -640 + 8 + 10 \\
|
||||
A & = & -632 + 10 \\
|
||||
A & = & -622
|
||||
\end{eqnarray*}
|
||||
\end{solution}
|
||||
|
||||
|
||||
\item $-7 x ( x - ( -7 ) )$ en $x = -10$
|
||||
|
||||
\item $-7 x ( x - ( -7 ) )$ en $x = -10$
|
||||
|
||||
|
||||
\item $( 8 x + 7 ) ( -5 - 2 x )$ en $x = -2$
|
||||
\end{enumerate}
|
||||
\end{multicols}
|
||||
\end{Exo}
|
||||
|
||||
\begin{Exo}
|
||||
Réduis les expressions suivantes
|
||||
|
||||
\begin{multicols}{3}
|
||||
\begin{enumerate}
|
||||
|
||||
\item A = $x + 1 + x - 4$
|
||||
\begin{solution}
|
||||
\begin{eqnarray*}
|
||||
A & = & x + 1 + x - 4 \\
|
||||
A & = & x + x + 1 - 4 \\
|
||||
A & = & ( 1 + 1 ) x + 1 - 4 \\
|
||||
A & = & 2 x - 3
|
||||
\end{eqnarray*}
|
||||
\end{solution}
|
||||
|
||||
\item B = $x + 6 + 3 + x - 6$
|
||||
\begin{solution}
|
||||
\begin{eqnarray*}
|
||||
A & = & x + 6 + 3 + x - 6 \\
|
||||
A & = & x + 9 + x - 6 \\
|
||||
A & = & x + x + 9 - 6 \\
|
||||
A & = & ( 1 + 1 ) x + 9 - 6 \\
|
||||
A & = & 2 x + 3
|
||||
\end{eqnarray*}
|
||||
\end{solution}
|
||||
|
||||
\item C = $-3 x + 5 + 5 x$
|
||||
\begin{solution}
|
||||
\begin{eqnarray*}
|
||||
A & = & -3 x + 5 + 5 x \\
|
||||
A & = & -3 x + 5 x + 5 \\
|
||||
A & = & ( -3 + 5 ) x + 5 \\
|
||||
A & = & 2 x + 5
|
||||
\end{eqnarray*}
|
||||
\end{solution}
|
||||
|
||||
|
||||
\item D = $-4 + 2 x - 10 - 10 x$
|
||||
\begin{solution}
|
||||
\begin{eqnarray*}
|
||||
A & = & -4 + 2 x - 10 - 10 x \\
|
||||
A & = & 2 x - 4 - 10 - 10 x \\
|
||||
A & = & 2 x - 14 - 10 x \\
|
||||
A & = & 2 x - 10 x - 14 \\
|
||||
A & = & ( 2 - 10 ) x - 14 \\
|
||||
A & = & -8 x - 14
|
||||
\end{eqnarray*}
|
||||
\end{solution}
|
||||
|
||||
\item E = $-1 - 8 + 8 x + 6 - 4 x$
|
||||
\begin{solution}
|
||||
\begin{eqnarray*}
|
||||
A & = & -1 - 8 + 8 x + 6 - 4 x \\
|
||||
A & = & -9 + 8 x + 6 - 4 x \\
|
||||
A & = & 8 x - 9 + 6 - 4 x \\
|
||||
A & = & 8 x - 3 - 4 x \\
|
||||
A & = & 8 x - 4 x - 3 \\
|
||||
A & = & ( 8 - 4 ) x - 3 \\
|
||||
A & = & 4 x - 3
|
||||
\end{eqnarray*}
|
||||
\end{solution}
|
||||
|
||||
\item E = $x^{ 2 } + 3 + 3 x + 3 - 10 x$
|
||||
\begin{solution}
|
||||
\begin{eqnarray*}
|
||||
A & = & x^{ 2 } + 3 + 3 x + 3 - 10 x \\
|
||||
A & = & x^{ 2 } + 3 x + 3 + 3 - 10 x \\
|
||||
A & = & x^{ 2 } + 3 x + 6 - 10 x \\
|
||||
A & = & x^{ 2 } + 3 x - 10 x + 6 \\
|
||||
A & = & x^{ 2 } + ( 3 - 10 ) x + 6 \\
|
||||
A & = & x^{ 2 } - 7 x + 6
|
||||
\end{eqnarray*}
|
||||
\end{solution}
|
||||
|
||||
|
||||
\item F = $-6 x^{ 2 } + 9 - 2 x - 2 - 6 x$
|
||||
\begin{solution}
|
||||
\begin{eqnarray*}
|
||||
A & = & -6 x^{ 2 } + 9 - 2 x - 2 - 6 x \\
|
||||
A & = & -6 x^{ 2 } - 2 x + 9 - 2 - 6 x \\
|
||||
A & = & -6 x^{ 2 } - 2 x + 7 - 6 x \\
|
||||
A & = & -6 x^{ 2 } - 2 x - 6 x + 7 \\
|
||||
A & = & -6 x^{ 2 } + ( -2 - 6 ) x + 7 \\
|
||||
A & = & -6 x^{ 2 } - 8 x + 7
|
||||
\end{eqnarray*}
|
||||
\end{solution}
|
||||
|
||||
\item G = $3 x^{ 2 } + 1 x^{ 2 } - 9 x - 9 + 3 x$
|
||||
\begin{solution}
|
||||
\begin{eqnarray*}
|
||||
A & = & 3 x^{ 2 } + 1 x^{ 2 } - 9 x - 9 + 3 x \\
|
||||
A & = & 3 x^{ 2 } + x^{ 2 } - 9 x - 9 + 3 x \\
|
||||
A & = & ( 3 + 1 ) x^{ 2 } - 9 x - 9 + 3 x \\
|
||||
A & = & 4 x^{ 2 } - 9 x - 9 + 3 x \\
|
||||
A & = & 4 x^{ 2 } - 9 x + 3 x - 9 \\
|
||||
A & = & 4 x^{ 2 } + ( -9 + 3 ) x - 9 \\
|
||||
A & = & 4 x^{ 2 } - 6 x - 9
|
||||
\end{eqnarray*}
|
||||
\end{solution}
|
||||
|
||||
\item G = $-2 x^{ 2 } + 7 - 6 x - 6 x^{ 2 } - 2 x$
|
||||
\begin{solution}
|
||||
\begin{eqnarray*}
|
||||
A & = & -2 x^{ 2 } + 7 - 6 x - 6 x^{ 2 } - 2 x \\
|
||||
A & = & -2 x^{ 2 } - 6 x + 7 - 6 x^{ 2 } - 2 x \\
|
||||
A & = & -2 x^{ 2 } - 6 x^{ 2 } - 6 x + 7 - 2 x \\
|
||||
A & = & ( -2 - 6 ) x^{ 2 } - 6 x + 7 - 2 x \\
|
||||
A & = & -8 x^{ 2 } - 6 x + 7 - 2 x \\
|
||||
A & = & -8 x^{ 2 } - 6 x - 2 x + 7 \\
|
||||
A & = & -8 x^{ 2 } + ( -6 - 2 ) x + 7 \\
|
||||
A & = & -8 x^{ 2 } - 8 x + 7
|
||||
\end{eqnarray*}
|
||||
\end{solution}
|
||||
\end{enumerate}
|
||||
\end{multicols}
|
||||
\end{Exo}
|
||||
}
|
||||
|
||||
\begin{document}
|
||||
|
||||
\lesexos
|
||||
\setcounter{exo}{0}
|
||||
\vfill
|
||||
|
||||
\ifprintanswers
|
||||
\else
|
||||
\lesexos
|
||||
\setcounter{exo}{0}
|
||||
\vfill
|
||||
\lesexos
|
||||
\setcounter{exo}{0}
|
||||
\vfill
|
||||
\fi
|
||||
|
||||
\end{document}
|
||||
|
||||
%%% Local Variables:
|
||||
%%% mode: latex
|
||||
%%% TeX-master: "master"
|
||||
%%% End:
|
Binary file not shown.
Binary file not shown.
BIN
3e/Expression_litterale/Periode2/all_eval_red_tech.pdf
Normal file
BIN
3e/Expression_litterale/Periode2/all_eval_red_tech.pdf
Normal file
Binary file not shown.
BIN
3e/Expression_litterale/Periode2/corr_eval_red_tech.pdf
Normal file
BIN
3e/Expression_litterale/Periode2/corr_eval_red_tech.pdf
Normal file
Binary file not shown.
BIN
3e/Expression_litterale/Periode2/formuleEuler.odt
Normal file
BIN
3e/Expression_litterale/Periode2/formuleEuler.odt
Normal file
Binary file not shown.
BIN
3e/Expression_litterale/Periode2/formuleEuler.pdf
Normal file
BIN
3e/Expression_litterale/Periode2/formuleEuler.pdf
Normal file
Binary file not shown.
BIN
3e/Expression_litterale/Periode2/formuleEulerpyramides.odt
Normal file
BIN
3e/Expression_litterale/Periode2/formuleEulerpyramides.odt
Normal file
Binary file not shown.
42
3e/Expression_litterale/Periode2/index.rst
Normal file
42
3e/Expression_litterale/Periode2/index.rst
Normal file
@@ -0,0 +1,42 @@
|
||||
Notes autour de la 3e période sur le calcul littéral pour les 3e: développer réduire
|
||||
####################################################################################
|
||||
|
||||
:date: 2016-10-06
|
||||
:modified: 2016-10-06
|
||||
:tags: Calcul litteral
|
||||
:category: 3e
|
||||
:authors: Bertrand Benjamin
|
||||
:summary: Organisation de la période 2 pour le calcul littéral pour les 3e
|
||||
|
||||
|
||||
Séance 1
|
||||
--------
|
||||
|
||||
Problèmes formule d'Euler (réduction d'expressions)
|
||||
|
||||
Séance 2
|
||||
--------
|
||||
|
||||
Exercices techniques de réduction
|
||||
|
||||
Séance 3
|
||||
--------
|
||||
|
||||
Programmes de calculs avec d'étranges coïncidences à explicité grâce au calcul littéral.
|
||||
|
||||
Séance 4
|
||||
--------
|
||||
|
||||
Programmes avec développements implicites.
|
||||
|
||||
Explication de la technique de développement
|
||||
|
||||
Exercices techniques de developpement.
|
||||
|
||||
Séances suivantes
|
||||
-----------------
|
||||
|
||||
Mélange de programmes de calculs et exercices techniques.
|
||||
|
||||
|
||||
|
BIN
3e/Expression_litterale/Periode2/pgm_cal_vers_expr_litt.odt
Normal file
BIN
3e/Expression_litterale/Periode2/pgm_cal_vers_expr_litt.odt
Normal file
Binary file not shown.
BIN
3e/Expression_litterale/Periode2/pgm_calc_vers_developpement.odt
Normal file
BIN
3e/Expression_litterale/Periode2/pgm_calc_vers_developpement.odt
Normal file
Binary file not shown.
BIN
3e/Expression_litterale/Periode2/pgm_calc_vers_developpement.pdf
Normal file
BIN
3e/Expression_litterale/Periode2/pgm_calc_vers_developpement.pdf
Normal file
Binary file not shown.
129
3e/Expression_litterale/Periode2/pgm_calc_vers_developpement.tex
Normal file
129
3e/Expression_litterale/Periode2/pgm_calc_vers_developpement.tex
Normal file
@@ -0,0 +1,129 @@
|
||||
\documentclass[a4paper,10pt,landscape, twocolumn]{/media/documents/Cours/Prof/Enseignements/2016-2017/tools/style/classExo}
|
||||
\usepackage{/media/documents/Cours/Prof/Enseignements/2016-2017/theme}
|
||||
%\geometry{left=10mm,right=10mm, top=10mm, bottom=10mm}
|
||||
|
||||
% Title Page
|
||||
\titre{Programme de calcul vers developpement}
|
||||
% \seconde \premiereS \PSTMG \TSTMG
|
||||
\classe{Troisième}
|
||||
\date{Décembre 2016}
|
||||
%\duree{1 heure}
|
||||
%\sujet{01}
|
||||
% DS DSCorr DM DMCorr Corr
|
||||
|
||||
%\printanswers
|
||||
\pagestyle{empty}
|
||||
|
||||
\newcommand{\lesexos}{%
|
||||
\begin{Exo}
|
||||
Voici 2 programmes de cacluls.
|
||||
|
||||
\Ovalbox{%
|
||||
\begin{minipage}{0.2\textwidth}
|
||||
\hspace{1cm}\textbf{Programme A}
|
||||
\begin{itemize}
|
||||
\item Choisir un nombre
|
||||
\item Multiplier par 4
|
||||
\item Soustraire 1
|
||||
\item Ajouter le nombre de départ
|
||||
\item Soustraire 2
|
||||
\end{itemize}
|
||||
\end{minipage}
|
||||
}
|
||||
\Ovalbox{%
|
||||
\begin{minipage}{0.2\textwidth}
|
||||
\hspace{1cm} \textbf{Programme B}
|
||||
\begin{itemize}
|
||||
\item Choisir un nombre
|
||||
\item Multiplier par 5
|
||||
\item Enlever 3
|
||||
\end{itemize}
|
||||
\end{minipage}
|
||||
}
|
||||
\begin{enumerate}
|
||||
\item Executer ces 2 programmes avec 4 nombres différents.
|
||||
\item Abdou pense
|
||||
\begin{center}
|
||||
\textit{Ces 2 programmes donnent toujours le même résultat.}
|
||||
\end{center}
|
||||
Qu'en pensez vous?
|
||||
\end{enumerate}
|
||||
\end{Exo}
|
||||
|
||||
\begin{Exo}
|
||||
\Ovalbox{%
|
||||
\begin{minipage}{0.2\textwidth}
|
||||
\hspace{1cm}\textbf{Programme de calcul}
|
||||
\begin{itemize}
|
||||
\item Choisir un nombre
|
||||
\item Multiplier par 2
|
||||
\item Ajouter 3
|
||||
\item Multiplier par 5
|
||||
\item Enlever 5 fois le nombre de départ
|
||||
\end{itemize}
|
||||
\end{minipage}
|
||||
}
|
||||
\begin{minipage}{0.2\textwidth}
|
||||
\begin{enumerate}
|
||||
\item Executer ce programme avec 4 nombres différents et faire une conjecture.
|
||||
\item Démontrer votre conjecture.
|
||||
\end{enumerate}
|
||||
\end{minipage}
|
||||
\end{Exo}
|
||||
|
||||
\begin{Exo}
|
||||
Voici 2 programmes de cacluls.
|
||||
|
||||
\hspace{1cm}
|
||||
\Ovalbox{%
|
||||
\begin{minipage}{0.2\textwidth}
|
||||
\hspace{1cm}\textbf{Programme A}
|
||||
\begin{itemize}
|
||||
\item Choisir un nombre
|
||||
\item Soustraire 1
|
||||
\item Multiplier par 4
|
||||
\item Ajouter le nombre de départ
|
||||
\item Soustraire 2
|
||||
\end{itemize}
|
||||
\end{minipage}
|
||||
}
|
||||
\hfill
|
||||
\Ovalbox{%
|
||||
\begin{minipage}{0.2\textwidth}
|
||||
\hspace{1cm} \textbf{Programme B}
|
||||
\begin{itemize}
|
||||
\item Choisir un nombre
|
||||
\item Soustraire 2
|
||||
\item Multiplier par 3
|
||||
\item Ajouter 2 fois le nombre de départ
|
||||
\end{itemize}
|
||||
\end{minipage}
|
||||
}
|
||||
\hspace{1cm}
|
||||
|
||||
\begin{enumerate}
|
||||
\item Executer ces 2 programmes avec 4 nombres différents.
|
||||
\item Quelle conjecture peut-on faire?
|
||||
\item Prouver la conjecture.
|
||||
\end{enumerate}
|
||||
\end{Exo}
|
||||
}
|
||||
|
||||
\begin{document}
|
||||
|
||||
\lesexos
|
||||
\setcounter{exo}{0}
|
||||
\vfill
|
||||
|
||||
\pagebreak
|
||||
|
||||
\lesexos
|
||||
\setcounter{exo}{0}
|
||||
\vfill
|
||||
|
||||
\end{document}
|
||||
|
||||
%%% Local Variables:
|
||||
%%% mode: latex
|
||||
%%% TeX-master: "master"
|
||||
%%% End:
|
179
3e/Expression_litterale/Periode2/tpl_eval_red_tech.tex
Normal file
179
3e/Expression_litterale/Periode2/tpl_eval_red_tech.tex
Normal file
@@ -0,0 +1,179 @@
|
||||
\documentclass[a4paper,10pt]{/media/documents/Cours/Prof/Enseignements/2016-2017/tools/style/classExo}
|
||||
\usepackage{/media/documents/Cours/Prof/Enseignements/2016-2017/theme}
|
||||
%\geometry{left=10mm,right=10mm, top=10mm, bottom=10mm}
|
||||
|
||||
% Title Page
|
||||
\titre{Technique: Evaluer et reduire}
|
||||
% \seconde \premiereS \PSTMG \TSTMG
|
||||
\classe{Troisième}
|
||||
\date{Novembre 2016}
|
||||
%\duree{1 heure}
|
||||
\sujet{\Var{infos.num}}
|
||||
% DS DSCorr DM DMCorr Corr
|
||||
|
||||
%\printanswers
|
||||
\pagestyle{empty}
|
||||
|
||||
\newcommand{\lesexos}{%
|
||||
\begin{Exo}
|
||||
Évaluer les expressions suivantes
|
||||
|
||||
\begin{multicols}{3}
|
||||
\begin{enumerate}
|
||||
\Block{set P = Polynom.random(degree = 1)}
|
||||
\Block{set x = Expression.random("{a}")}
|
||||
\item $\Var{P}$ en $x = \Var{x}$
|
||||
\begin{solution}
|
||||
\begin{eqnarray*}
|
||||
\Var{P(x).explain()| calculus()}
|
||||
\end{eqnarray*}
|
||||
\end{solution}
|
||||
\Block{set P = Polynom.random(degree = 1)}
|
||||
\Block{set x = Expression.random("{a}")}
|
||||
\item $\Var{P}$ en $x = \Var{x}$
|
||||
\begin{solution}
|
||||
\begin{eqnarray*}
|
||||
\Var{P(x).explain()| calculus()}
|
||||
\end{eqnarray*}
|
||||
\end{solution}
|
||||
\Block{set x = Expression.random("{a}")}
|
||||
\item $\Var{P}$ en $x = \Var{x}$
|
||||
\begin{solution}
|
||||
\begin{eqnarray*}
|
||||
\Var{P(x).explain()| calculus()}
|
||||
\end{eqnarray*}
|
||||
\end{solution}
|
||||
|
||||
\Block{set P = Polynom.random(degree = 2)}
|
||||
\Block{set x = Expression.random("{a}")}
|
||||
\item $\Var{P}$ en $x = \Var{x}$
|
||||
\begin{solution}
|
||||
\begin{eqnarray*}
|
||||
\Var{P(x).explain()| calculus()}
|
||||
\end{eqnarray*}
|
||||
\end{solution}
|
||||
\Block{set x = Expression.random("{a}")}
|
||||
\item $\Var{P}$ en $x = \Var{x}$
|
||||
\begin{solution}
|
||||
\begin{eqnarray*}
|
||||
\Var{P(x).explain()| calculus()}
|
||||
\end{eqnarray*}
|
||||
\end{solution}
|
||||
\Block{set P = Polynom.random(degree = 2)}
|
||||
\Block{set x = Expression.random("{a}")}
|
||||
\item $\Var{P}$ en $x = \Var{x}$
|
||||
\begin{solution}
|
||||
\begin{eqnarray*}
|
||||
\Var{P(x).explain()| calculus()}
|
||||
\end{eqnarray*}
|
||||
\end{solution}
|
||||
\Block{set P = Expression.random("{a} x * (x - {b})")}
|
||||
\Block{set x = Expression.random("{a}")}
|
||||
\item $\Var{P}$ en $x = \Var{x}$
|
||||
\Block{set x = Expression.random("{a}")}
|
||||
\item $\Var{P}$ en $x = \Var{x}$
|
||||
\Block{set P = Expression.random("({a}x + {b})({c} + {d}x)")}
|
||||
\Block{set x = Expression.random("{a}")}
|
||||
\item $\Var{P}$ en $x = \Var{x}$
|
||||
\end{enumerate}
|
||||
\end{multicols}
|
||||
\end{Exo}
|
||||
|
||||
\begin{Exo}
|
||||
Réduis les expressions suivantes
|
||||
|
||||
\begin{multicols}{3}
|
||||
\begin{enumerate}
|
||||
\Block{set A = Expression.random("x + {a} + x - {b}")}
|
||||
\item A = $\Var{A}$
|
||||
\begin{solution}
|
||||
\begin{eqnarray*}
|
||||
\Var{A.simplify().explain() | calculus()}
|
||||
\end{eqnarray*}
|
||||
\end{solution}
|
||||
\Block{set B = Expression.random("x + {a} + {c} + x - {b}")}
|
||||
\item B = $\Var{B}$
|
||||
\begin{solution}
|
||||
\begin{eqnarray*}
|
||||
\Var{B.simplify().explain() | calculus()}
|
||||
\end{eqnarray*}
|
||||
\end{solution}
|
||||
\Block{set C = Expression.random("{a}x + {c} + {d}x", conditions = ["{a} not in [0,1]", "{d} not in [0,1]"])}
|
||||
\item C = $\Var{C}$
|
||||
\begin{solution}
|
||||
\begin{eqnarray*}
|
||||
\Var{C.simplify().explain() | calculus()}
|
||||
\end{eqnarray*}
|
||||
\end{solution}
|
||||
|
||||
\Block{set D = Expression.random("{b} + {a}x + {c} + {d}x", conditions = ["{a} not in [0,1]", "{d} not in [0,1]"])}
|
||||
\item D = $\Var{D}$
|
||||
\begin{solution}
|
||||
\begin{eqnarray*}
|
||||
\Var{D.simplify().explain() | calculus()}
|
||||
\end{eqnarray*}
|
||||
\end{solution}
|
||||
\Block{set E = Expression.random("({b} - {a}) + {a}x + {c} + {d}x", conditions = ["{a} not in [0,1]", "{d} not in [0,1]"])}
|
||||
\item E = $\Var{E}$
|
||||
\begin{solution}
|
||||
\begin{eqnarray*}
|
||||
\Var{E.simplify().explain() | calculus()}
|
||||
\end{eqnarray*}
|
||||
\end{solution}
|
||||
\Block{set E = Expression.random("x^2 + {c} + {d}x + {d} + {a}x", conditions = ["{a} not in [0,1]", "{d} not in [0,1]"])}
|
||||
\item E = $\Var{E}$
|
||||
\begin{solution}
|
||||
\begin{eqnarray*}
|
||||
\Var{E.simplify().explain() | calculus()}
|
||||
\end{eqnarray*}
|
||||
\end{solution}
|
||||
|
||||
\Block{set F = Expression.random("{a}x^2 + {c} + {d}x + {d} + {a}x", conditions = ["{a} not in [0,1]", "{d} not in [0,1]"])}
|
||||
\item F = $\Var{F}$
|
||||
\begin{solution}
|
||||
\begin{eqnarray*}
|
||||
\Var{F.simplify().explain() | calculus()}
|
||||
\end{eqnarray*}
|
||||
\end{solution}
|
||||
\Block{set G = Expression.random("{a}x^2 + {c}x^2 + {d}x + {d} + {a}x", conditions = ["{a} not in [0,1]", "{d} not in [0,1]"])}
|
||||
\item G = $\Var{G}$
|
||||
\begin{solution}
|
||||
\begin{eqnarray*}
|
||||
\Var{G.simplify().explain() | calculus()}
|
||||
\end{eqnarray*}
|
||||
\end{solution}
|
||||
\Block{set G = Expression.random("{a}x^2 + {c}+ {d}x + {d}x^2 + {a}x", conditions = ["{a} not in [0,1]", "{d} not in [0,1]"])}
|
||||
\item G = $\Var{G}$
|
||||
\begin{solution}
|
||||
\begin{eqnarray*}
|
||||
\Var{G.simplify().explain() | calculus()}
|
||||
\end{eqnarray*}
|
||||
\end{solution}
|
||||
\end{enumerate}
|
||||
\end{multicols}
|
||||
\end{Exo}
|
||||
}
|
||||
|
||||
\begin{document}
|
||||
|
||||
\lesexos
|
||||
\setcounter{exo}{0}
|
||||
\vfill
|
||||
|
||||
\ifprintanswers
|
||||
\else
|
||||
\lesexos
|
||||
\setcounter{exo}{0}
|
||||
\vfill
|
||||
\lesexos
|
||||
\setcounter{exo}{0}
|
||||
\vfill
|
||||
\fi
|
||||
|
||||
\end{document}
|
||||
|
||||
%%% Local Variables:
|
||||
%%% mode: latex
|
||||
%%% TeX-master: "master"
|
||||
%%% End:
|
||||
|
Binary file not shown.
Binary file not shown.
BIN
3e/Expression_litterale/Periode3/Exercice des murs.odt
Normal file
BIN
3e/Expression_litterale/Periode3/Exercice des murs.odt
Normal file
Binary file not shown.
41
3e/Expression_litterale/Periode3/index.rst
Normal file
41
3e/Expression_litterale/Periode3/index.rst
Normal file
@@ -0,0 +1,41 @@
|
||||
Notes autour de la 3e période sur le calcul littéral pour les 3e: découverte des équations
|
||||
##########################################################################################
|
||||
|
||||
:date: 2016-11-26
|
||||
:modified: 2016-11-26
|
||||
:tags: Calcul litteral, Equation
|
||||
:category: 3e
|
||||
:authors: Bertrand Benjamin
|
||||
:summary: 3e période pour les 3e sur le calcul littéral. On y retravaille développer réduire et on attaque des équations.
|
||||
|
||||
Étapes 1, 2 (et 3?)
|
||||
-------------------
|
||||
|
||||
Rituel: exo techniques développement (à balancer au tableau)
|
||||
|
||||
Exercices d'introduction (calculatrice et tableur) aux équations `Trouver un nombre inconnu <./nombre_inconu.pdf>`_
|
||||
|
||||
Quelques égalités à trous pour s'assurer du sens du signe "=" et pour commencer à construire des techniques de résolutions.
|
||||
|
||||
Étapes 3
|
||||
--------
|
||||
|
||||
Les égalités à trous nous amène à remplacer les trous par une lettre et de formaliser (et expliquer) les techniques personnelles.
|
||||
|
||||
On peut alors définir une équation et faire décrire les techniques sur le cahier de bord.
|
||||
|
||||
Résolution d'équation technique
|
||||
|
||||
Remarque
|
||||
Il faudrait revoir cette approche qui ne marche pas si bien que ça. En s'appuyant, plus sur la mise en équation.
|
||||
|
||||
Étapes 4, 5, 6
|
||||
--------------
|
||||
|
||||
Rituel: exercices techniques avec des équations
|
||||
|
||||
Problèmes qui peuvent être mis en équations
|
||||
|
||||
- `Avec des inconnues d'un seul côté <./pb_equation_inconnue_dun_cote.pdf>`_
|
||||
- `Avec des inconnues des deux côtés <./pb_equation_inconnue_2_cotes.pdf>`_
|
||||
|
BIN
3e/Expression_litterale/Periode3/nombre_inconu.pdf
Normal file
BIN
3e/Expression_litterale/Periode3/nombre_inconu.pdf
Normal file
Binary file not shown.
101
3e/Expression_litterale/Periode3/nombre_inconu.tex
Normal file
101
3e/Expression_litterale/Periode3/nombre_inconu.tex
Normal file
@@ -0,0 +1,101 @@
|
||||
\documentclass[a4paper,10pt]{/media/documents/Cours/Prof/Enseignements/2016-2017/tools/style/classExo}
|
||||
\usepackage{/media/documents/Cours/Prof/Enseignements/2016-2017/theme}
|
||||
|
||||
% Title Page
|
||||
\titre{Initiation aux équations - Exercices}
|
||||
% \seconde \premiereS \PSTMG \TSTMG
|
||||
\classe{Troisième}
|
||||
\date{Janvier 2017}
|
||||
|
||||
\renewcommand{\arraystretch}{1.3}
|
||||
|
||||
\newcommand{\lexo}{%
|
||||
|
||||
\begin{Exo}[Calculatrice]
|
||||
Emma et Zoé ont chacune une calculatrice. Elles on "tapé" le même nombre.
|
||||
|
||||
Ensuite, Emma a appuyé sur les touches
|
||||
|
||||
\begin{tabular}{|*{5}{p{1cm}|}}
|
||||
\hline
|
||||
$\times$ & 2 & $+$ & 3 & EXE ($=$)\\
|
||||
\hline
|
||||
\end{tabular}
|
||||
|
||||
et Zoé a appuyé sur les touches
|
||||
|
||||
\begin{tabular}{|*{8}{p{1cm}|}}
|
||||
\hline
|
||||
$-$ & 2 & EXE ($=$) & $\times$ & 4 & $+$ & 8 &EXE ($=$) \\
|
||||
\hline
|
||||
\end{tabular}
|
||||
|
||||
Quelle coïncidence: elles obtiennent le même résultat! Quel nombre ont-elles bien pu choisir?
|
||||
\end{Exo}
|
||||
|
||||
\begin{Exo}[Programme de calculs]
|
||||
\Ovalbox{%
|
||||
\begin{minipage}{0.3\textwidth}
|
||||
\hspace{0.5cm}Programme A
|
||||
\begin{itemize}
|
||||
\item Choisir un nombre
|
||||
\item Multiplier par 5
|
||||
\item Ajouter 4
|
||||
\end{itemize}
|
||||
\end{minipage}
|
||||
}
|
||||
\hspace{1cm}
|
||||
\Ovalbox{%
|
||||
\begin{minipage}{0.3\textwidth}
|
||||
\hspace{0.5cm}Programme B
|
||||
\begin{itemize}
|
||||
\item Choisir un nombre
|
||||
\item Ajouter 2
|
||||
\item Multiplier par 6
|
||||
\item Soustraire 5
|
||||
\end{itemize}
|
||||
\end{minipage}
|
||||
}
|
||||
|
||||
\begin{enumerate}
|
||||
\item J'ai effectué le programme A et j'ai trouvé 19 comme résultat. Quel nobre ai-je choisi?
|
||||
\item J'ai executé le programme B et j'ai trouvé 88 comme résultat. Quel nombre ai-je choisi?
|
||||
\item Même question avec le nombre 5.
|
||||
\end{enumerate}
|
||||
\end{Exo}
|
||||
|
||||
\begin{Exo}[Tableur]
|
||||
Abdou et Mariam travaillent sur un même ordinateur et ont ouvert une feuille de calcul (tableur).
|
||||
|
||||
\begin{itemize}
|
||||
\item Dans la cellule \Ovalbox{\texttt{A1}} ils s'amusent à mettre des nombres au hasard.
|
||||
\item Dans la cellule \Ovalbox{\texttt{B1}} Abdou a rentré la formule \Ovalbox{\texttt{=A1*6+7}}.
|
||||
\item Dans la cellule \Ovalbox{\texttt{C1}} Mariam a rentré la formule \Ovalbox{\texttt{=2*(A1+11)}}.
|
||||
\end{itemize}
|
||||
|
||||
Au bout d'un moment, ils s'apperçoivent que \texttt{B1=C1}. Quel nombre ont-ils mis dans la cellule \texttt{A1}?
|
||||
\end{Exo}
|
||||
|
||||
}
|
||||
|
||||
\geometry{left=15mm,right=15mm, bottom= 15mm, top=15mm}
|
||||
|
||||
\pagestyle{empty}
|
||||
|
||||
\begin{document}
|
||||
|
||||
\lexo
|
||||
|
||||
\setcounter{exo}{0}
|
||||
\vfill
|
||||
|
||||
\lexo
|
||||
|
||||
|
||||
\end{document}
|
||||
|
||||
%%% Local Variables:
|
||||
%%% mode: latex
|
||||
%%% TeX-master: "master"
|
||||
%%% End:
|
||||
|
Binary file not shown.
Binary file not shown.
Binary file not shown.
Binary file not shown.
BIN
3e/Expression_litterale/Periode4/QCM.pdf
Normal file
BIN
3e/Expression_litterale/Periode4/QCM.pdf
Normal file
Binary file not shown.
74
3e/Expression_litterale/Periode4/QCM.tex
Normal file
74
3e/Expression_litterale/Periode4/QCM.tex
Normal file
@@ -0,0 +1,74 @@
|
||||
\documentclass[a5paper,10pt]{/media/documents/Cours/Prof/Enseignements/2016-2017/tools/style/classExo}
|
||||
\usepackage{/media/documents/Cours/Prof/Enseignements/2016-2017/theme}
|
||||
|
||||
% Title Page
|
||||
\titre{Calcul litteral et équation- Exercices}
|
||||
% \seconde \premiereS \PSTMG \TSTMG
|
||||
\classe{Troisième}
|
||||
\date{Mars 2017}
|
||||
|
||||
\newcommand{\lexo}{%
|
||||
\setcounter{exo}{0}
|
||||
\begin{Exo}[QCM]
|
||||
Cet exercice est un QCM (questionnaire à choix multiples). Pour chaque ligne du tableau, une seule affirmation est juste. Sur votre copie, indiquer le numéro de la question et recopier l’affirmation juste.
|
||||
|
||||
On ne demande pas de justifier.
|
||||
|
||||
\vfill
|
||||
|
||||
\noindent
|
||||
\begin{tabular}{|c|p{4cm}|*{3}{p{2cm}|}}
|
||||
\hline
|
||||
&& Réponse A & Réponse B & Réponse C \\
|
||||
\hline
|
||||
1& $-4x(-x+3) = ...$ & $7$ & $4x^2-12x$ & $4x^2+12x$ \\
|
||||
\hline
|
||||
2& $4x + 2x(x + 3)= ...$ & $9x$ & $6x + 6$ & $2x^2 + 10x$ \\
|
||||
\hline
|
||||
3& Une solution de \newline $7x - 21 =0$ est & 0 & 3 & 4 \\
|
||||
\hline
|
||||
4& Une solution de \newline $3x - 11 =2x$ est & 11 & 2 & Impossible \\
|
||||
\hline
|
||||
5&3 appliqué à \newline
|
||||
\Ovalbox{%
|
||||
\parbox{2.8cm}{
|
||||
Choisir un nombre \newline
|
||||
Mettre au carré \newline
|
||||
Enlever 10
|
||||
}} donne
|
||||
& $-1$ & $1$ & Impossible\\
|
||||
\hline
|
||||
6&x appliqué à \newline
|
||||
\Ovalbox{%
|
||||
\parbox{2.8cm}{
|
||||
Choisir un nombre \newline
|
||||
Doubler \newline
|
||||
Soustraire 4
|
||||
}} donne
|
||||
& $x^2 - 4$ & $2(x-3)$ & $2x - 3$\\
|
||||
\hline
|
||||
|
||||
\end{tabular}
|
||||
\end{Exo}
|
||||
|
||||
|
||||
}
|
||||
|
||||
%\geometry{left=15mm,right=15mm, bottom= 15mm, top=15mm}
|
||||
|
||||
\pagestyle{empty}
|
||||
|
||||
\begin{document}
|
||||
|
||||
\lexo
|
||||
|
||||
\vfill
|
||||
|
||||
|
||||
\end{document}
|
||||
|
||||
%%% Local Variables:
|
||||
%%% mode: latex
|
||||
%%% TeX-master: "master"
|
||||
%%% End:
|
||||
|
BIN
3e/Expression_litterale/Periode4/QCM_308.pdf
Normal file
BIN
3e/Expression_litterale/Periode4/QCM_308.pdf
Normal file
Binary file not shown.
74
3e/Expression_litterale/Periode4/QCM_308.tex
Normal file
74
3e/Expression_litterale/Periode4/QCM_308.tex
Normal file
@@ -0,0 +1,74 @@
|
||||
\documentclass[a5paper,10pt]{/media/documents/Cours/Prof/Enseignements/2016-2017/tools/style/classExo}
|
||||
\usepackage{/media/documents/Cours/Prof/Enseignements/2016-2017/theme}
|
||||
|
||||
% Title Page
|
||||
\titre{Calcul litteral et équation- Exercices}
|
||||
% \seconde \premiereS \PSTMG \TSTMG
|
||||
\classe{Troisième}
|
||||
\date{Mars 2017}
|
||||
|
||||
\newcommand{\lexo}{%
|
||||
\setcounter{exo}{0}
|
||||
\begin{Exo}[QCM]
|
||||
Cet exercice est un QCM (questionnaire à choix multiples). Pour chaque ligne du tableau, une seule affirmation est juste. Sur votre copie, indiquer le numéro de la question et recopier l’affirmation juste.
|
||||
|
||||
On ne demande pas de justifier.
|
||||
|
||||
\vfill
|
||||
|
||||
\noindent
|
||||
\begin{tabular}{|c|p{4cm}|*{3}{p{2cm}|}}
|
||||
\hline
|
||||
&& Réponse A & Réponse B & Réponse C \\
|
||||
\hline
|
||||
1& $4x - 4 + 3x + 14= ...$ & $17x$ & $7x + 10$ & $1x + 18$ \\
|
||||
\hline
|
||||
2& $9x + 3 - 11x - 3= ...$ & $-2x$ & $20x - 6$ & $11x$ \\
|
||||
\hline
|
||||
3& Une solution de \newline $7x - 21 =0$ est & 0 & 3 & 4 \\
|
||||
\hline
|
||||
4& Une solution de \newline $3x - 11 =2x$ est & 11 & 2 & Impossible \\
|
||||
\hline
|
||||
5&3 appliqué à \newline
|
||||
\Ovalbox{%
|
||||
\parbox{2.8cm}{
|
||||
Choisir un nombre \newline
|
||||
Mettre au carré \newline
|
||||
Enlever 10
|
||||
}} donne
|
||||
& $-1$ & $1$ & Impossible\\
|
||||
\hline
|
||||
6&6 appliqué à \newline
|
||||
\Ovalbox{%
|
||||
\parbox{2.8cm}{
|
||||
Choisir un nombre \newline
|
||||
Doubler \newline
|
||||
Soustraire 4
|
||||
}} donne
|
||||
& $6^2 - 4$ & $2(6-4)$ & $2\times6 - 4$\\
|
||||
\hline
|
||||
|
||||
\end{tabular}
|
||||
\end{Exo}
|
||||
|
||||
|
||||
}
|
||||
|
||||
%\geometry{left=15mm,right=15mm, bottom= 15mm, top=15mm}
|
||||
|
||||
\pagestyle{empty}
|
||||
|
||||
\begin{document}
|
||||
|
||||
\lexo
|
||||
|
||||
\vfill
|
||||
|
||||
|
||||
\end{document}
|
||||
|
||||
%%% Local Variables:
|
||||
%%% mode: latex
|
||||
%%% TeX-master: "master"
|
||||
%%% End:
|
||||
|
BIN
3e/Expression_litterale/Periode4/QCM_308_a4.pdf
Normal file
BIN
3e/Expression_litterale/Periode4/QCM_308_a4.pdf
Normal file
Binary file not shown.
BIN
3e/Expression_litterale/Periode4/QCM_a4.pdf
Normal file
BIN
3e/Expression_litterale/Periode4/QCM_a4.pdf
Normal file
Binary file not shown.
BIN
3e/Expression_litterale/Periode4/eq_tech.pdf
Normal file
BIN
3e/Expression_litterale/Periode4/eq_tech.pdf
Normal file
Binary file not shown.
145
3e/Expression_litterale/Periode4/eq_tech.tex
Normal file
145
3e/Expression_litterale/Periode4/eq_tech.tex
Normal file
@@ -0,0 +1,145 @@
|
||||
\documentclass[a5paper,12pt]{/media/documents/Cours/Prof/Enseignements/2016-2017/tools/style/classExo}
|
||||
\usepackage{/media/documents/Cours/Prof/Enseignements/2016-2017/theme}
|
||||
|
||||
% Title Page
|
||||
\titre{Équation du premier degré - Exercices}
|
||||
% \seconde \premiereS \PSTMG \TSTMG
|
||||
\classe{Troisième}
|
||||
\date{Mars 2017}
|
||||
|
||||
\geometry{left=10mm,right=10mm, bottom= 10mm, top=10mm}
|
||||
\pagestyle{empty}
|
||||
%\printanswers
|
||||
|
||||
\begin{document}
|
||||
|
||||
\begin{Exo}
|
||||
Résoudre les équations suivantes
|
||||
\begin{multicols}{2}
|
||||
\begin{enumerate}
|
||||
\item $x + 79 = 82$
|
||||
\begin{solution}
|
||||
$x = 82 - 79 = 3$
|
||||
\end{solution}
|
||||
\item $x + 23 = 17$
|
||||
\begin{solution}
|
||||
~ $x = 17 - 23 = -6$
|
||||
\end{solution}
|
||||
\item $a - 32 = 10$
|
||||
\begin{solution}
|
||||
~ $a = 10 + 32 = 42$
|
||||
\end{solution}
|
||||
\item $14x = 37$
|
||||
\begin{solution}
|
||||
~ $x = \frac{37}{14} = 2.642857142857143$
|
||||
\end{solution}
|
||||
\item $20y = 18$
|
||||
\begin{solution}
|
||||
~ $y = \frac{18}{20} = 0.9$
|
||||
\end{solution}
|
||||
\item $x + 10 = 24$
|
||||
\begin{solution}
|
||||
$x = 24 - 10 = 14$
|
||||
\end{solution}
|
||||
\item $x + 41 = 7$
|
||||
\begin{solution}
|
||||
~ $x = 7 - 41 = -34$
|
||||
\end{solution}
|
||||
\item $a - 80 = 29$
|
||||
\begin{solution}
|
||||
~ $a = 29 + 80 = 109$
|
||||
\end{solution}
|
||||
\item $80x = 57$
|
||||
\begin{solution}
|
||||
~ $x = \frac{57}{80} = 0.7125$
|
||||
\end{solution}
|
||||
\item $57y = 95$
|
||||
\begin{solution}
|
||||
~ $y = \frac{95}{57} = 1.6666666666666667$
|
||||
\end{solution}
|
||||
\end{enumerate}
|
||||
\end{multicols}
|
||||
\end{Exo}
|
||||
|
||||
\begin{Exo}
|
||||
Résoudre les équations suivantes
|
||||
\begin{multicols}{2}
|
||||
\begin{enumerate}
|
||||
\item $9x + 84 = 0$
|
||||
\begin{solution}
|
||||
$x = \frac{0 - 84}{9} = -9.333333333333334$
|
||||
\end{solution}
|
||||
\item $4x + 71 = 14$
|
||||
\begin{solution}
|
||||
$x = \frac{14 - 71}{4} = -14.25$
|
||||
\end{solution}
|
||||
\item $10x + 87 = 71$
|
||||
\begin{solution}
|
||||
$x = \frac{71 - 87}{10} = -1.6$
|
||||
\end{solution}
|
||||
\item $5x + 25 = 2x + 17$
|
||||
\begin{solution}
|
||||
$x = \frac{17 - 25}{5 - 2} = -2.6666666666666665$
|
||||
\end{solution}
|
||||
\item $8x + 79 = 6x + 68$
|
||||
\begin{solution}
|
||||
$x = \frac{68 - 79}{8 - 6} = -5.5$
|
||||
\end{solution}
|
||||
\item $4x - 61 = 0$
|
||||
\begin{solution}
|
||||
$x = \frac{0 + 61}{4} = 15.25$
|
||||
\end{solution}
|
||||
\item $5x + 68 = 30$
|
||||
\begin{solution}
|
||||
$x = \frac{30 - 68}{5} = -7.6$
|
||||
\end{solution}
|
||||
\item $5x + 64 = 12x + 93$
|
||||
\begin{solution}
|
||||
$x = \frac{93 - 64}{5 - 12} = -4.142857142857143$
|
||||
\end{solution}
|
||||
\item $3x + 77 = 7x + 16$
|
||||
\begin{solution}
|
||||
$x = \frac{16 - 77}{3 - 7} = 15.25$
|
||||
\end{solution}
|
||||
\item $4x + 20 = -7x + 89$
|
||||
\begin{solution}
|
||||
$x = \frac{89 - 20}{4 - -7} = 6.2727272727272725$
|
||||
\end{solution}
|
||||
\end{enumerate}
|
||||
\end{multicols}
|
||||
\end{Exo}
|
||||
|
||||
\begin{Exo}
|
||||
Voici deux programmes de calculs
|
||||
|
||||
\fbox{\colorbox{base2}{
|
||||
\begin{minipage}[h]{0.4\textwidth}
|
||||
\textbf{Programme A} \\
|
||||
Choisir un nombre \\
|
||||
Multiplier par 5 \\
|
||||
Ajouter 3
|
||||
\end{minipage}
|
||||
}
|
||||
}
|
||||
\fbox{\colorbox{base2}{
|
||||
\begin{minipage}[h]{0.4\textwidth}
|
||||
\textbf{Programme B} \\
|
||||
Choisir un nombre \\
|
||||
Doubler \\
|
||||
Enlever 10
|
||||
\end{minipage}
|
||||
}
|
||||
}
|
||||
\begin{enumerate}
|
||||
\item Est-ce que ces deux programmes donnent toujours le même résultat?
|
||||
\item Quelle valeur faut-il choisir pour obtenir 3 pour chaque programme? \textit{On demande de trouver ce résultat avec une équation}
|
||||
\item Trouver la valeur de départ pour que ces deux programmes donnent le même résultat.
|
||||
\end{enumerate}
|
||||
\end{Exo}
|
||||
|
||||
\end{document}
|
||||
|
||||
%%% Local Variables:
|
||||
%%% mode: latex
|
||||
%%% TeX-master: "master"
|
||||
%%% End:
|
BIN
3e/Expression_litterale/Periode4/eq_tech_a4.pdf
Normal file
BIN
3e/Expression_litterale/Periode4/eq_tech_a4.pdf
Normal file
Binary file not shown.
BIN
3e/Expression_litterale/Periode4/factorisation_relier.pdf
Normal file
BIN
3e/Expression_litterale/Periode4/factorisation_relier.pdf
Normal file
Binary file not shown.
75
3e/Expression_litterale/Periode4/factorisation_relier.tex
Normal file
75
3e/Expression_litterale/Periode4/factorisation_relier.tex
Normal file
@@ -0,0 +1,75 @@
|
||||
\documentclass[a5paper,12pt]{/media/documents/Cours/Prof/Enseignements/2016-2017/tools/style/classExo}
|
||||
\usepackage{/media/documents/Cours/Prof/Enseignements/2016-2017/theme}
|
||||
|
||||
% Title Page
|
||||
\titre{Factorisation - Exercices}
|
||||
% \seconde \premiereS \PSTMG \TSTMG
|
||||
\classe{Troisième}
|
||||
\date{Avril 2017}
|
||||
|
||||
|
||||
\pagestyle{empty}
|
||||
\geometry{left=5mm,right=5mm, bottom= 10mm, top=10mm}
|
||||
|
||||
\renewcommand{\lexo}{%
|
||||
\setcounter{exo}{0}
|
||||
\begin{Exo}
|
||||
\begin{enumerate}
|
||||
\item Relier les expressions égales entres elles.
|
||||
|
||||
\begin{minipage}[c]{0.4\textwidth}
|
||||
\flushright
|
||||
$3x + 9 \qquad \bullet$ \\[0.5cm]
|
||||
$2x^2 + 3x \qquad \bullet$ \\[0.5cm]
|
||||
$3x^2 + 9x \qquad \bullet$ \\[0.5cm]
|
||||
$25x^2 + 5x \qquad \bullet$
|
||||
|
||||
\end{minipage}
|
||||
\hspace{2cm}
|
||||
\begin{minipage}[c]{0.4\textwidth}
|
||||
\begin{itemize}
|
||||
\item $5(x+5)$
|
||||
\item $x(2x+3)$
|
||||
\item $5x(5x+1)$
|
||||
\item $x(3x+9)$
|
||||
\item $3(x+9)$
|
||||
\item $3x(x^2 +3)$
|
||||
\item $3(x+3)$
|
||||
\item $x(x+3)$
|
||||
\end{itemize}
|
||||
|
||||
\end{minipage}
|
||||
|
||||
\item Factoriser les expressions suivantes
|
||||
\begin{multicols}{2}
|
||||
\begin{enumerate}
|
||||
\item $4x + 6$
|
||||
\item $3x^2 + 2x$
|
||||
\item $25x^2 + 16x$
|
||||
\item $2x + 5x^2$
|
||||
\end{enumerate}
|
||||
|
||||
\end{multicols}
|
||||
|
||||
\end{enumerate}
|
||||
\end{Exo}
|
||||
}
|
||||
|
||||
|
||||
\begin{document}
|
||||
|
||||
\lexo
|
||||
|
||||
\hfill
|
||||
|
||||
\lexo
|
||||
|
||||
|
||||
|
||||
\end{document}
|
||||
|
||||
%%% Local Variables:
|
||||
%%% mode: latex
|
||||
%%% TeX-master: "master"
|
||||
%%% End:
|
||||
|
BIN
3e/Expression_litterale/Periode4/factorisation_relier2.pdf
Normal file
BIN
3e/Expression_litterale/Periode4/factorisation_relier2.pdf
Normal file
Binary file not shown.
81
3e/Expression_litterale/Periode4/factorisation_relier2.tex
Normal file
81
3e/Expression_litterale/Periode4/factorisation_relier2.tex
Normal file
@@ -0,0 +1,81 @@
|
||||
\documentclass[a4paper,12pt,landscape, twocolumn]{/media/documents/Cours/Prof/Enseignements/2016-2017/tools/style/classExo}
|
||||
\usepackage{/media/documents/Cours/Prof/Enseignements/2016-2017/theme}
|
||||
|
||||
% Title Page
|
||||
\titre{Factorisation - Exercices}
|
||||
% \seconde \premiereS \PSTMG \TSTMG
|
||||
\classe{Troisième}
|
||||
\date{Mars 2017}
|
||||
|
||||
|
||||
\pagestyle{empty}
|
||||
\geometry{left=5mm,right=5mm, bottom= 10mm, top=10mm}
|
||||
|
||||
|
||||
\newcommand{\exoEnCours}{
|
||||
\begin{Exo}
|
||||
\begin{enumerate}
|
||||
\item Relier les expressions égales entres elles. Puis écrire les égalités trouvées.
|
||||
|
||||
\begin{minipage}[c]{0.2\textwidth}
|
||||
\flushright
|
||||
$6x + 12 \qquad \bullet$ \\[0.5cm]
|
||||
$3x^2 + 10x \qquad \bullet$ \\[0.5cm]
|
||||
$15x^2 + 5x \qquad \bullet$ \\[0.5cm]
|
||||
$14x^2 + 7 \qquad \bullet$
|
||||
|
||||
\end{minipage}
|
||||
\hspace{2cm}
|
||||
\begin{minipage}[c]{0.1\textwidth}
|
||||
\begin{itemize}
|
||||
\item $x(3x+10)$
|
||||
\item $15x(x+1)$
|
||||
\item $7(2x^2+1)$
|
||||
\item $3x(x+10)$
|
||||
\item $x(15x+5)$
|
||||
\item $6(x+2)$
|
||||
\item $5x(3x+1)$
|
||||
\end{itemize}
|
||||
|
||||
\end{minipage}
|
||||
|
||||
\item Factoriser les expressions suivantes
|
||||
\begin{multicols}{2}
|
||||
\begin{enumerate}
|
||||
\item $9x + 27$
|
||||
\item $15x^2 + 2x$
|
||||
\item $14x^2 + 2x$
|
||||
\item $10x + 7x^2$
|
||||
\end{enumerate}
|
||||
|
||||
\end{multicols}
|
||||
\end{enumerate}
|
||||
\end{Exo}
|
||||
}
|
||||
|
||||
\begin{document}
|
||||
|
||||
\exoEnCours
|
||||
|
||||
\setcounter{exo}{0}
|
||||
|
||||
\exoEnCours
|
||||
\setcounter{exo}{0}
|
||||
|
||||
\pagebreak
|
||||
|
||||
\exoEnCours
|
||||
|
||||
\setcounter{exo}{0}
|
||||
|
||||
\exoEnCours
|
||||
|
||||
\pagebreak
|
||||
|
||||
\end{document}
|
||||
|
||||
%%% Local Variables:
|
||||
%%% mode: latex
|
||||
%%% TeX-master: "master"
|
||||
%%% End:
|
||||
|
BIN
3e/Expression_litterale/Periode4/factorisation_relier_a4.pdf
Normal file
BIN
3e/Expression_litterale/Periode4/factorisation_relier_a4.pdf
Normal file
Binary file not shown.
43
3e/Expression_litterale/Periode4/index.rst
Normal file
43
3e/Expression_litterale/Periode4/index.rst
Normal file
@@ -0,0 +1,43 @@
|
||||
Notes autour de la 4e période sur le calcul littéral pour les 3e: équation et factorisation
|
||||
###########################################################################################
|
||||
|
||||
:date: 2017-03-14
|
||||
:modified: 2017-03-14
|
||||
:tags: Calcul litteral, QCM
|
||||
:category: 3e
|
||||
:authors: Bertrand Benjamin
|
||||
:summary: 4e période pour les 3e sur le calcul littéral. On y retravaille les équations et on découvre la factorisation.
|
||||
|
||||
Étapes 1
|
||||
--------
|
||||
|
||||
Initiation aux méthodes liées au QCM. C'est l'occasion de revoir tout ce qui a été vu en calcul littéral depuis le début de l'année.
|
||||
|
||||
On commence par un `petit QCM <./intro_QCM.pdf>`_ projeté au tableau les élèves y répondent seuls, on corrige en expliquant comment répondre et quels sont les stratégies que l'on peut mettre en place.
|
||||
|
||||
Puis on continue avec un `plus grand QCM <./QCM.pdf>`_ distribué aux élèves. Avec ce QCM est fourni le traditionnel petit texte pour que les élèves sachent comment répondre. On répondra à ce QCM en 2 étapes:
|
||||
- Les réponses sont données au hasard à partir de lancés de dé
|
||||
- Les élèves écrivent la réponse qu'ils pensent juste.
|
||||
|
||||
On distribue et on compte les points de chacun.
|
||||
|
||||
Étape 2 à 4
|
||||
-----------
|
||||
|
||||
On travaille en parallèle la résolution d'équation et la factorisation.
|
||||
|
||||
Pour les équations, on travaille d'abord la technique puis on revient vers les problèmes données à la période précédente
|
||||
- `Exercices techniques de résolution d'équations <./eq_tech.pdf>`_
|
||||
- `Problèmes avec résolution d'équations <../Periode3/pb_equation_inconnue_dun_cote.pdf>`_
|
||||
|
||||
Pour la factorisation, on s'appuie sur ce que l'on connait sur le développement pour voir le problème à l'envers.
|
||||
- `relier les expressions égales <./factorisation_relier.pdf>`_
|
||||
- `fiche d'exercices techniques sur la factorisation.`
|
||||
|
||||
Étape 5 à 7
|
||||
-----------
|
||||
|
||||
(je n'arrive pas à comprendre si c'est encore au programme..!) On utilise la factorisation pour résoudre des équations de degrés 2.
|
||||
|
||||
|
||||
|
BIN
3e/Expression_litterale/Periode4/intro_QCM.pdf
Normal file
BIN
3e/Expression_litterale/Periode4/intro_QCM.pdf
Normal file
Binary file not shown.
46
3e/Expression_litterale/Periode4/intro_QCM.tex
Executable file
46
3e/Expression_litterale/Periode4/intro_QCM.tex
Executable file
@@ -0,0 +1,46 @@
|
||||
\documentclass[a4paper,10pt,xcolor=table]{/media/documents/Cours/Prof/Enseignements/2016-2017/tools/style/classPres}
|
||||
\usepackage{/media/documents/Cours/Prof/Enseignements/2016-2017/theme}
|
||||
|
||||
\author{}
|
||||
\title{}
|
||||
\date{}
|
||||
|
||||
\begin{document}
|
||||
|
||||
\begin{frame}{Consignes du QCM}
|
||||
|
||||
Cet exercice est un questionnaire à choix multiple (QCM). Pour chaque ligne du tableau, trois réponses sont proposées, mais une seule est exacte.
|
||||
|
||||
Toute réponse exacte vaut 1 point.
|
||||
|
||||
Toute réponse inexacte ou toute absence de réponse n’enlève pas de point.
|
||||
|
||||
|
||||
Indiquez sur votre copie le numéro de la question et, sans justifier, recopier la réponse exacte (A ou B ou C).
|
||||
|
||||
\end{frame}
|
||||
|
||||
\begin{frame}{QCM}
|
||||
\begin{tabular}{|c|p{3cm}|*{3}{m{2cm}|}}
|
||||
\hline
|
||||
&& Réponse A & Réponse B & Réponse C \\
|
||||
\hline
|
||||
1&$3x - 2x(x-1) = ...$ & $2x^2+3x-1$ & $-2x^2-3x+1$ & $-2x^2+3x+1$ \\
|
||||
\hline
|
||||
2&Une solution de \newline $x^2-2x-8=0$ est & 0 & 3 & 4 \\
|
||||
\hline
|
||||
3&x appliqué à \newline
|
||||
\Ovalbox{%
|
||||
\parbox{2.8cm}{
|
||||
Choisir un nombre \newline
|
||||
Ajoute 3 \newline
|
||||
Multiplier par 2
|
||||
}} donne
|
||||
& $2(x+3)$ & $x+3\times2$ & $2x - 6$\\
|
||||
\hline
|
||||
\end{tabular}
|
||||
\end{frame}
|
||||
|
||||
|
||||
|
||||
\end{document}
|
BIN
3e/Expression_litterale/Periode4/intro_QCM_308.pdf
Normal file
BIN
3e/Expression_litterale/Periode4/intro_QCM_308.pdf
Normal file
Binary file not shown.
46
3e/Expression_litterale/Periode4/intro_QCM_308.tex
Executable file
46
3e/Expression_litterale/Periode4/intro_QCM_308.tex
Executable file
@@ -0,0 +1,46 @@
|
||||
\documentclass[a4paper,10pt,xcolor=table]{/media/documents/Cours/Prof/Enseignements/2016-2017/tools/style/classPres}
|
||||
\usepackage{/media/documents/Cours/Prof/Enseignements/2016-2017/theme}
|
||||
|
||||
\author{}
|
||||
\title{}
|
||||
\date{}
|
||||
|
||||
\begin{document}
|
||||
|
||||
\begin{frame}{Consignes du QCM}
|
||||
|
||||
Cet exercice est un questionnaire à choix multiple (QCM). Pour chaque ligne du tableau, trois réponses sont proposées, mais une seule est exacte.
|
||||
|
||||
Toute réponse exacte vaut 1 point.
|
||||
|
||||
Toute réponse inexacte ou toute absence de réponse n’enlève pas de point.
|
||||
|
||||
|
||||
Indiquez sur votre copie le numéro de la question et, sans justifier, recopier la réponse exacte (A ou B ou C).
|
||||
|
||||
\end{frame}
|
||||
|
||||
\begin{frame}{QCM}
|
||||
\begin{tabular}{|c|p{3cm}|*{3}{m{2cm}|}}
|
||||
\hline
|
||||
&& Réponse A & Réponse B & Réponse C \\
|
||||
\hline
|
||||
1&$6x - 3x + 12 + 2 = ...$ & $17x$ & $17$ & $3x + 14$ \\
|
||||
\hline
|
||||
2&Une solution de \newline $3x - 15=0$ est & 2 & 3 & 5 \\
|
||||
\hline
|
||||
3&4 appliqué à \newline
|
||||
\Ovalbox{%
|
||||
\parbox{2.8cm}{
|
||||
Choisir un nombre \newline
|
||||
Ajoute 3 \newline
|
||||
Multiplier par 2
|
||||
}} donne
|
||||
& $(4+3)\times2$ & $4+3\times2$ & $10$\\
|
||||
\hline
|
||||
\end{tabular}
|
||||
\end{frame}
|
||||
|
||||
|
||||
|
||||
\end{document}
|
BIN
3e/Expression_litterale/Periode5/QCM_308.pdf
Normal file
BIN
3e/Expression_litterale/Periode5/QCM_308.pdf
Normal file
Binary file not shown.
75
3e/Expression_litterale/Periode5/QCM_308.tex
Executable file
75
3e/Expression_litterale/Periode5/QCM_308.tex
Executable file
@@ -0,0 +1,75 @@
|
||||
\documentclass[a5paper,10pt]{/media/documents/Cours/Prof/Enseignements/2016-2017/tools/style/classExo}
|
||||
\usepackage{/media/documents/Cours/Prof/Enseignements/2016-2017/theme}
|
||||
|
||||
% Title Page
|
||||
\titre{Calcul litteral et équation- Exercices}
|
||||
% \seconde \premiereS \PSTMG \TSTMG
|
||||
\classe{Troisième}
|
||||
\date{Mars 2017}
|
||||
|
||||
\newcommand{\lexo}{%
|
||||
\setcounter{exo}{0}
|
||||
\begin{Exo}[QCM]
|
||||
Cet exercice est un QCM (questionnaire à choix multiples). Pour chaque ligne du tableau, une seule affirmation est juste. Sur votre copie, indiquer le numéro de la question et recopier l’affirmation juste.
|
||||
|
||||
On ne demande pas de justifier.
|
||||
|
||||
\vfill
|
||||
|
||||
\noindent
|
||||
\begin{tabular}{|c|p{4cm}|*{3}{p{2cm}|}}
|
||||
\hline
|
||||
&& Réponse A & Réponse B & Réponse C \\
|
||||
\hline
|
||||
1& $5x - 3 + 2x + 6= ...$ & $8$ & $7x + 3$ & $3x + 9$ \\
|
||||
\hline
|
||||
2& $6 + 2x - 4x - 5= ...$ & $-1x$ & $11 - 2x$ & $1 - 2x$ \\
|
||||
\hline
|
||||
3&9 appliqué à \newline
|
||||
\Ovalbox{%
|
||||
\parbox{2.8cm}{
|
||||
Choisir un nombre \newline
|
||||
Doubler\newline
|
||||
Enlever 10
|
||||
}} donne
|
||||
& $8$ & $2$ & Impossible\\
|
||||
\hline
|
||||
4&Le nombre qui appliqué à \newline
|
||||
\Ovalbox{%
|
||||
\parbox{2.8cm}{
|
||||
Choisir un nombre \newline
|
||||
Multiplier par 3 \newline
|
||||
Soustraire 4
|
||||
}} donne 10 est
|
||||
& $4,666$ & $\dfrac{14}{6}$ & N'existe pas \\
|
||||
\hline
|
||||
5& Une solution de \newline $7x = 21 $ est & 0 & 3 & 4 \\
|
||||
\hline
|
||||
6& $x > 3$ signifie que & $x$ est plus grand que 3 & $x$ est plus petit que 3 & $x$ est égal à 3 \\
|
||||
\hline
|
||||
7& Si $x < 4$ alors & $x$ peut être égal à 2 & $x$ peut être égal à 4 & $x$ peut être égal à 10 \\
|
||||
\hline
|
||||
\end{tabular}
|
||||
\end{Exo}
|
||||
|
||||
|
||||
}
|
||||
|
||||
%\geometry{left=15mm,right=15mm, bottom= 15mm, top=15mm}
|
||||
|
||||
\pagestyle{empty}
|
||||
|
||||
\begin{document}
|
||||
|
||||
\lexo
|
||||
|
||||
\vfill
|
||||
|
||||
|
||||
\end{document}
|
||||
|
||||
%%% Local Variables:
|
||||
%%% mode: latex
|
||||
%%% TeX-master: "master"
|
||||
%%% End:
|
||||
|
BIN
3e/Expression_litterale/Periode5/QCM_308_a4.pdf
Normal file
BIN
3e/Expression_litterale/Periode5/QCM_308_a4.pdf
Normal file
Binary file not shown.
BIN
3e/Expression_litterale/Periode5/QCM_312.pdf
Normal file
BIN
3e/Expression_litterale/Periode5/QCM_312.pdf
Normal file
Binary file not shown.
92
3e/Expression_litterale/Periode5/QCM_312.tex
Executable file
92
3e/Expression_litterale/Periode5/QCM_312.tex
Executable file
@@ -0,0 +1,92 @@
|
||||
\documentclass[a5paper,10pt]{/media/documents/Cours/Prof/Enseignements/2016-2017/tools/style/classExo}
|
||||
\usepackage{/media/documents/Cours/Prof/Enseignements/2016-2017/theme}
|
||||
|
||||
% Title Page
|
||||
\titre{Calcul litteral et équation- Exercices}
|
||||
% \seconde \premiereS \PSTMG \TSTMG
|
||||
\classe{Troisième}
|
||||
\date{Mars 2017}
|
||||
|
||||
\newcommand{\lexo}{%
|
||||
\setcounter{exo}{0}
|
||||
\begin{Exo}[QCM]
|
||||
Cet exercice est un QCM (questionnaire à choix multiples). Pour chaque ligne du tableau, une seule affirmation est juste. Sur votre copie, indiquer le numéro de la question et recopier l’affirmation juste.
|
||||
|
||||
On ne demande pas de justifier.
|
||||
|
||||
\vfill
|
||||
|
||||
\noindent
|
||||
\begin{tabular}{|c|p{4cm}|*{3}{p{2cm}|}}
|
||||
\hline
|
||||
&& Réponse A & Réponse B & Réponse C \\
|
||||
\hline
|
||||
1& $(2x+3)(5x-2) = ...$ & $10x^2+19x-6$ & $10x^2 - 6$ & $7x + 1$ \\
|
||||
\hline
|
||||
2& $6x^2 - 3x= ...$ & $3x^2$ & $3x(2x+1)$ & $x(6x-3)$ \\
|
||||
\hline
|
||||
3& $(3x - 1)^2 = ...$ & $9x^2 - 1$ & $9x^2 + 1$ & $9x^2 - 6x + 1$ \\
|
||||
\hline
|
||||
4& Une solution de \newline $7x + 1 = 4x + 22 $ est & -1 & 7 & 10 \\
|
||||
\hline
|
||||
5& $x > 3$ signifie que & $x$ est plus grand que 3 & $x$ est plus petit que 3 & $x$ est égal à 3 \\
|
||||
\hline
|
||||
6& Si $x+2 > 5$ alors & $x$ peut être égal 0 & $x$ peut être égal à 3 & $x$ peut être égal à 10 \\
|
||||
\hline
|
||||
\end{tabular}
|
||||
\end{Exo}
|
||||
|
||||
\vfill
|
||||
|
||||
\begin{Exo}[Revenons aux programmes]
|
||||
Voici 2 programmes de cacluls.
|
||||
|
||||
\Ovalbox{%
|
||||
\begin{minipage}{0.4\textwidth}
|
||||
\hspace{1cm}\textbf{Programme A}
|
||||
\begin{itemize}
|
||||
\item Choisir un nombre
|
||||
\item Multiplier par 7
|
||||
\item Soustraire 2
|
||||
\end{itemize}
|
||||
\end{minipage}
|
||||
}
|
||||
\hfill
|
||||
\Ovalbox{%
|
||||
\begin{minipage}{0.4\textwidth}
|
||||
\hspace{1cm} \textbf{Programme B}
|
||||
\begin{itemize}
|
||||
\item Choisir un nombre
|
||||
\item Ajouter 2
|
||||
\item Multiplier par 3
|
||||
\item Enlever 4
|
||||
\end{itemize}
|
||||
\end{minipage}
|
||||
}
|
||||
\begin{enumerate}
|
||||
\item Pour chacun des programmes, quel nombre doit-on choisir pour avoir 20 à la fin? Et 15?
|
||||
\item Quel nombre doit-on choisir pour que les 2 programmes donnent le même résultat?
|
||||
\end{enumerate}
|
||||
|
||||
\end{Exo}
|
||||
\vfill
|
||||
}
|
||||
|
||||
%\geometry{left=15mm,right=15mm, bottom= 15mm, top=15mm}
|
||||
|
||||
\pagestyle{empty}
|
||||
|
||||
\begin{document}
|
||||
|
||||
\lexo
|
||||
|
||||
\vfill
|
||||
|
||||
|
||||
\end{document}
|
||||
|
||||
%%% Local Variables:
|
||||
%%% mode: latex
|
||||
%%% TeX-master: "master"
|
||||
%%% End:
|
||||
|
BIN
3e/Expression_litterale/Periode5/QCM_312_a4.pdf
Normal file
BIN
3e/Expression_litterale/Periode5/QCM_312_a4.pdf
Normal file
Binary file not shown.
BIN
3e/Expression_litterale/Periode5/devTech_ineqPb.pdf
Normal file
BIN
3e/Expression_litterale/Periode5/devTech_ineqPb.pdf
Normal file
Binary file not shown.
97
3e/Expression_litterale/Periode5/devTech_ineqPb.tex
Executable file
97
3e/Expression_litterale/Periode5/devTech_ineqPb.tex
Executable file
@@ -0,0 +1,97 @@
|
||||
\documentclass[a5paper,10pt]{/media/documents/Cours/Prof/Enseignements/2016-2017/tools/style/classExo}
|
||||
\usepackage{/media/documents/Cours/Prof/Enseignements/2016-2017/theme}
|
||||
|
||||
% Title Page
|
||||
\titre{Calcul litteral et inéquation- Exercices}
|
||||
% \seconde \premiereS \PSTMG \TSTMG
|
||||
\classe{Troisième}
|
||||
\date{Juin 2017}
|
||||
|
||||
\newcommand{\lexo}{%
|
||||
\setcounter{exo}{0}
|
||||
\begin{Exo}[Developper]
|
||||
Développer et réduire les expressions suivantes
|
||||
\begin{multicols}{3}
|
||||
\begin{enumerate}
|
||||
\item $3x(-x+2)$
|
||||
\item $(3x-2)(x+1)$
|
||||
|
||||
\item $(3x-2)(x+1)$
|
||||
\item $(2x+3)^2$
|
||||
|
||||
\item $3x(-x+2)$
|
||||
\item $(3x-2)(x+1)$
|
||||
\end{enumerate}
|
||||
\end{multicols}
|
||||
|
||||
\end{Exo}
|
||||
\vfill
|
||||
\begin{Exo}[Factoriser]
|
||||
Factoriser les expressions suivantes
|
||||
\begin{multicols}{2}
|
||||
\begin{enumerate}
|
||||
\item $48x^2 - 16x$
|
||||
\item $(x+1)(3x-1) + (x+1)(5x + 2)$
|
||||
|
||||
\item $(2x+3)(-x+1) + (5x+1)(2x + 3)$
|
||||
\item $(x+1)(3x-1) - (x+1)(5x + 2)$
|
||||
\end{enumerate}
|
||||
\end{multicols}
|
||||
|
||||
\end{Exo}
|
||||
\vfill
|
||||
\begin{Exo}[Longueur des batons]
|
||||
\begin{minipage}{0.7\textwidth}
|
||||
Trois bâtons mesurent ensemble plus de 2,5 m.
|
||||
|
||||
Le 2 ème mesure 0,3 m de plus que le premier.
|
||||
|
||||
Le 3 ème mesure 0,2 m de moins que le premier.
|
||||
|
||||
Quelle sont toutes les longueurs possibles de chaque bâton ?
|
||||
|
||||
\end{minipage}
|
||||
\begin{minipage}{0.3\textwidth}
|
||||
\includegraphics[scale=0.6]{./fig/batons}
|
||||
\end{minipage}
|
||||
|
||||
\end{Exo}
|
||||
\vfill
|
||||
\begin{Exo}[Hauteur d'un étage]
|
||||
\begin{minipage}{0.6\textwidth}
|
||||
Un immeuble de 4 étages mesure moins de 17,6 m de hauteur.
|
||||
La hauteur du toit mesure 1,5 fois la hauteur d'un étage.
|
||||
|
||||
Quelle sont les hauteurs possibles des étages?
|
||||
\end{minipage}
|
||||
\begin{minipage}{0.4\textwidth}
|
||||
\includegraphics[scale=0.7]{./fig/immeuble}
|
||||
\end{minipage}
|
||||
\end{Exo}
|
||||
\vfill
|
||||
\begin{Exo}[Courses]
|
||||
Pour la rentrée scolaire Blandine achète 6 classeurs et un livre. Elle a payé moins de 30€.
|
||||
|
||||
Le prix du livre est 12 €. Quel est le prix maximal d'un classeur ?
|
||||
|
||||
\end{Exo}
|
||||
}
|
||||
|
||||
%\geometry{left=15mm,right=15mm, bottom= 15mm, top=15mm}
|
||||
|
||||
\pagestyle{empty}
|
||||
|
||||
\begin{document}
|
||||
|
||||
\lexo
|
||||
|
||||
\vfill
|
||||
|
||||
|
||||
\end{document}
|
||||
|
||||
%%% Local Variables:
|
||||
%%% mode: latex
|
||||
%%% TeX-master: "master"
|
||||
%%% End:
|
||||
|
BIN
3e/Expression_litterale/Periode5/devTech_ineqPb_a4.pdf
Normal file
BIN
3e/Expression_litterale/Periode5/devTech_ineqPb_a4.pdf
Normal file
Binary file not shown.
BIN
3e/Expression_litterale/Periode5/fig/batons.pdf
Normal file
BIN
3e/Expression_litterale/Periode5/fig/batons.pdf
Normal file
Binary file not shown.
BIN
3e/Expression_litterale/Periode5/fig/immeuble.pdf
Normal file
BIN
3e/Expression_litterale/Periode5/fig/immeuble.pdf
Normal file
Binary file not shown.
36
3e/Expression_litterale/Periode5/index.rst
Normal file
36
3e/Expression_litterale/Periode5/index.rst
Normal file
@@ -0,0 +1,36 @@
|
||||
Notes autour de la 5e période sur le calcul littéral pour les 3e: Identités remarquables et inéquations
|
||||
#######################################################################################################
|
||||
|
||||
:date: 2017-04-25
|
||||
:modified: 2017-05-09
|
||||
:tags: Calcul litteral
|
||||
:category: 3e
|
||||
:authors: Bertrand Benjamin
|
||||
:summary: 5e période pour les 3e sur le calcul littéral. On y découvre les inéquations et les identités remarquables.
|
||||
|
||||
Étapes 1
|
||||
--------
|
||||
|
||||
On refait une série QCM pour se rappeler de la méthode à appliquer.
|
||||
|
||||
- `QCM pour les 308 <./QCM_308.pdf>`_
|
||||
|
||||
Étape 2
|
||||
-------
|
||||
|
||||
Découverte d'une première identité remarquable à travers un exercice à relier.
|
||||
|
||||
Entrainement aux équations.
|
||||
|
||||
Étape 3
|
||||
-------
|
||||
|
||||
Factorisation avec la première identité remarquable.
|
||||
|
||||
Initiations aux inéquations. Exercice qui mène à la résolution d'inéquation pour faire émerger l'objet inéquation.
|
||||
|
||||
Étape 4
|
||||
-------
|
||||
|
||||
Deuxième identité remarquable et approfondissement des inéquations.
|
||||
|
16
3e/Expression_litterale/index.rst
Normal file
16
3e/Expression_litterale/index.rst
Normal file
@@ -0,0 +1,16 @@
|
||||
Progression sur le calcul littéral en 3e pour l'année 2016-2017
|
||||
###############################################################
|
||||
|
||||
:date: 2017-03-14
|
||||
:modified: 2017-03-14
|
||||
:tags: Progression
|
||||
:category: 3e
|
||||
:authors: Bertrand Benjamin
|
||||
:summary: Progression sur le calcul littéral pour les 3e en 2016-2017
|
||||
|
||||
On réserve une heure par semaine à cette progression.
|
||||
|
||||
- `Période 1 <./Periode1/>`_
|
||||
- `Période 2 <./Periode2/>`_
|
||||
- `Période 3 <./Periode3/>`_
|
||||
- `Période 4 <./Periode4/>`_
|
Reference in New Issue
Block a user